You are on page 1of 62

G.R. No.

215957

COMMISSIONER OF INTERNAL REVENUE, Petitioner


vs.
FITNESS BY DESIGN, INC., Respondent

DECISION

LEONEN, J.:

To avail of the extraordinary period of assessment in Section 222(a) of the National Internal Revenue
Code, the Commissioner of Internal Revenue should show that the facts upon which the fraud' is based
is communicated to the taxpayer. The burden of proving that the facts exist in any subsequent
proceeding is with the Commissioner. Furthermore, the Final Assessment Notice is not valid if it does
not contain a definite due date for payment by the taxpayer.

This resolves a Petition for Review on Certiorari1 filed by the Commissioner of Internal Revenue, which
assails the Decision2 dated July 14, 2014 and Resolution3 dated December 16, 2014 of the Court of Tax
Appeals. The Court of Tax Appeals En Banc affirmed the Decision of the First Division, which declared the
assessment issued against Fitness by Design, Inc. (Fitness) as invalid.4

On April 11, 1996, Fitness filed its Annual Income Tax Return for the taxable year of 1995.5 According to
Fitness, it was still in its pre-operating stage during the covered period.6

On June 9, 2004, Fitness received a copy of the Final Assessment Notice dated March 17, 2004.7 The
Final Assessment Notice was issued under Letter of Authority No. 00002953.8 The Final Assessment
Notice assessed that Fitness had a tax deficiency in the amount of ₱10,647,529.69.9 It provides:

FINAL ASSESSMENT NOTICE

March 17, 2004

FITNESS BY DESIGN, INC


169 Aguirre St., BF Homes,
Paranaque City

Gentlemen:

Please be informed that after investigation of your Internal revenue Tax Liabilities for the year 1995
pursuant to Letter of Authority No. 000029353 dated May 13, 2002, there has been found due
deficiency taxes as shown hereunder:

Assessment No. _____________

Income Tax
Taxable Income per return ₱

Add: Unreported Sales 7,156,336.08

Taxable Income per audit 7,156,336.08

Tax Due (35%) 2,504,717.63

Add: Surcharge (50%) ₱ 1,252,358.81

Interest (20%/annum) until 4-15-04 4,508,491. 73 5, 760,850.54

Deficiency Income Tax ₱ 8,265,568.17

Value Added Tax

Unreported Sales ₱ 7,156,336.08

Output Tax (10%) 715,633.61

Add: Surcharge (50%) ₱ 357,816.80

Interest (20%/ annum) until 4-15-04 1,303,823.60 1,661,640.41

Deficiency VAT ₱ 2,311,214.02

Documentary Stamp Tax


Subscribe Capital Stock ₱ 375,000.00

DST due (2/200) 3,750.00

Add: Surcharge (25%) 937.50

Deficiency DST ₱ 4,687.50

Total Deficiency Taxes ₱ 10,647,529.69

The complete details covering the aforementioned discrepancies established during the investigation of
this case are shown in the accompanying Annex 1 of this Notice. The 50% surcharge and 20% interest
have been imposed pursuant to Sections 248 and 249(B) of the [National Internal Revenue Code], as
amended. Please note, however, that the interest and the total amount due will have to be adjusted if
paid prior or beyond April 15, 2004.

In view thereof, you are requested to pay your aforesaid deficiency internal revenue taxes liabilities
through the duly authorized agent bank in which you are enrolled within the time shown in the enclosed
assessment notice.10 (Emphasis in the original)

Fitness filed a protest to the Final Assessment Notice on June 25, 2004. According to Fitness, the
Commissioner's period to assess had already prescribed. Further, the assessment was without basis
since the company was only incorporated on May 30, 1995.11

On February 2, 2005, the Commissioner issued a Warrant of Distraint and/or Levy with Reference No.
OCN WDL-95-05-005 dated February 1, 2005 to Fitness.12

Fitness filed before the First Division of the Court of Tax Appeals a Petition for Review (With Motion to
Suspend Collection of Income Tax, Value Added Tax, Documentary Stamp Tax and Surcharges and
Interests) on March 1, 2005.13

On May 17, 2005, the Commissioner of Internal Revenue filed an Answer to Fitness' Petition and raised
special and affirmative defenses.14 The Commissioner posited that the Warrant of Distraint and/or Levy
was issued in accordance with law.15 The Commissioner claimed that its right to assess had not yet
prescribed under Section 222(a)16 of the National Internal Revenue Code.17 Because the 1995 Income
Tax ,Return filed by Fitness was false and fraudulent for its alleged intentional failure to reflect its true
sales, Fitness' respective taxes may be assessed at any time within 10 years from the discovery of fraud
or omission.18

The Commissioner asserted further that the assessment already became final and executory for Fitness'
failure , to file a protest within the reglementary period.19 The Commissioner denied that there was a
protest to the Final Assessment Notice filed by Fitness on June 25, 2004.20 According to the
Commissioner, the alleged protest was "nowhere to be found in the [Bureau of Internal Revenue]
Records nor reflected in the Record Book of the Legal Division as normally done by [its]' receiving clerk
when she received [sic] any document."21 Therefore, the Commissioner had sufficient basis to collect
the tax deficiency through the Warrant of Distraint and/or Levy.22

The alleged fraudulent return was discovered through a tip from a confidential informant.23 The
revenue officers' investigation revealed that Fitness had been operating business with sales operations
amounting to ₱7,156,336.08 in 1995, which it neglected toreport in its income tax return.24 Fitness'
failure to report its income resulted in deficiencies to its income tax and value-added tax of
₱8,265,568.17 and ₱2,377,274.02 respectively, as well as the documentary stamp tax with regard to
capital stock subscription.25

Through the report, the revenue officers recommended the filing of a civil case for collection of taxes
and a criminal case for failure to declare Fitness' purported sales in its 1995 Income Tax
Return.26 Hence, a criminal complaint against Fitness was filed before the Department of Justice.27

The Court of Tax Appeals First Division granted Fitness' Petition on the ground that the assessment has
already prescribed.28 It cancelled and set aside the Final Assessment Notice dated March 1 7, 2004 as
well as the Warrant of Distraint and/or Levy issued by the Commissioner.29 It ruled that the Final
Assessment Notice is invalid for failure to comply with the requirements of Section 22830 of the
National Internal Revenue Code. The dispositive portion of the Decision reads:

WHEREFORE, the Petition for Review dated February 24, 2005 filed by petitioner Fitness by Design, Inc.,
is hereby GRANTED. Accordingly, the Final Assessment Notice dated 'March 17, 2004, finding petitioner
liable for deficiency income tax, documentary stamp tax and value-added tax for taxable year 1995 in
the total amount of ₱10,647,529.69 is hereby CANCELLED and SET ASIDE. The Warrant of Distraint and
Levy dated February 1, 2005 is 'likewise CANCELLED and SET ASIDE.

SO ORDERED.31 (Emphasis in the original)

The Commissioner's Motion for Reconsideration and its Supplemental Motion for Reconsideration were
denied by the Court of Tax Appeals First Division.32

Aggrieved, the Commissioner filed an appeal before the Court of Tax Appeals En Banc.33 The
Commissioner asserted ,that it had 10 years to make an assessment due to the fraudulent income tax
return filed by Fitness.34 It also claimed that the assessment already attained finality due to Fitness'
failure to file its protest within the period provided by law.35

Fitness argued that the Final Assessment Notice issued to it could not be claimed as a valid deficiency
assessment that could justify the issuance of a warrant of distraint and/or levy.36 It asserted that it was
a mere request for payment as it did not provide the period within which to pay the alleged liabilities.37

The Court of Tax Appeals En Banc ruled in favor of Fitness. It affirmed the Decision of the Court of Tax
Appeals First Division, thus:
WHEREFORE, the instant Petition for Review is DENIED for lack of merit. Accordingly, both the Decision
and Resolution in CTA Case No. 7160 dated July 10, 2012 and November 21, 2012 respectively
are AFFIRMED in toto.38 (Emphasis in the original)

The Commissioner's Motion for Reconsideration was denied by the Court of Tax Appeals En Banc in the
Resolution39 dated December 16, 2014.

Hence, the Commissioner of Internal Revenue filed before this Court a Petition for Review.

Petitioner Commissioner of Internal Revenue raises the sole issue of whether the Final Assessment
Notice issued against respondent Fitness by Design, Inc. is a valid assessment under Section 228 of the
National Internal Revenue Code and Revenue Regulations No. 12-99.40

Petitioner argues that the Final Assessment Notice issued to respondent is valid since it complies with
Section 228 of the National Internal Revenue Code and Revenue Regulations No. 12-99.41 The law
states that the taxpayer shall be informed in writing of the facts, jurisprudence, and law on which the
assessment is based.42 Nothing in the law provides that due date for payment is a substantive
requirement for the validity of a final assessment notice.43

Petitioner further claims that a perusal of the Final Assessment Notice shows that April 15, 2004 is the
due date for payment.44 The pertinent portion of the assessment reads:

The complete details covering the aforementioned discrepancies established during the investigation of
this case are shown in the accompanying Annex 1 of this Notice. The 50% surcharge and 20% interest
have been imposed pursuant to Sections 248 and 249(B) of the [National Internal Revenue Code], as
amended. Please note, however, that the interest and the total amount due will have to be adjusted if
paid prior or beyond April 15, 2004.45 (Emphasis supplied)

This Court, through the Resolution46 dated July 22, 2015, required respondent to comment on the
Petition for Review.

In its Comment,47 respondent argues that the Final Assessment Notice issued was merely a request and
not a demand for payment of tax liabilities.48 The Final Assessment Notice cannot be considered as a
final deficiency assessment because it deprived respondent of due process when it failed to reflect its
fixed tax liabilities.49Moreover, it also gave respondent an indefinite period to pay its tax liabilities.50

Respondent points out that an assessment should strictly comply with the law for its
validity.51 Jurisprudence provides that "not all documents coming from the [Bureau of Internal
Revenue] containing a computation of the tax liability can be deemed assessments[,] which can attain
finality."52 Therefore, the Warrant of Distraint and/or Levy cannot be enforced since it is based on an
invalid assessment.53

Respondent likewise claims that since the Final Assessment Notice was allegedly based on fraud, it must
show the details of the fraudulent acts imputed to it as part of due process.54
I

The Petition has no merit.

An assessment "refers to the determination of amounts due from a person obligated to make
payments."55 "In the context of national internal revenue collection, it refers to the determination of
the taxes due from a taxpayer under the National Internal Revenue Code of 1997."56

The assessment process starts with the filing of tax return and payment of tax by the taxpayer.57 The
initial assessment evidenced by the tax return is a self-assessment of the taxpayer.58 The tax is primarily
computed and voluntarily paid by the taxpayer without need of any demand from government.59 If tax
obligations are properly paid, the Bureau of Internal Revenue may dispense with its own assessment.60

After filing a return, the Commissioner or his or her representative may allow the examination of any
taxpayer for assessment of proper tax liability.61 The failure of a taxpayer to file his or her return will
not hinder the Commissioner from permitting the taxpayer's examination.62 The Commissioner can
examine records or other data relevant to his or her inquiry in order to verify the correctness of any
return, or to make a return in case of noncompliance, as well as to determine and collect tax liability.63

The indispensability of affording taxpayers sufficient written notice of his or her tax liability is a clear
definite requirement.64 Section 228 of the National Internal Revenue Code and Revenue Regulations
No. 12-99, as amended, transparently outline the procedure in tax assessment.65

Section 3 of Revenue Regulations No. 12-99,66 the then prevailing regulation regarding the due process
requirement in the issuance of a deficiency tax assessment, requires a notice for informal
conference.67 The revenue officer who audited the taxpayer's records shall state in his or her report
whether the taxpayer concurs with his or her findings of liability for deficiency taxes.68 If the taxpayer
does not agree, based on the revenue officer's report, the taxpayer shall be informed in writing69 of the
discrepancies in his or her payment of internal revenue taxes for "Informal Conference."70 The informal
conference gives the taxpayer an opportunity to present his or her side of the case.71

The taxpayer is given 15 days from receipt of the notice of informal conference to respond.72 If the
taxpayer fails to respond, he or she will be considered in default.73 The revenue officer74 endorses the
case with the least possible delay to the Assessment Division of the Revenue Regional Office or the
Commissioner or his or her authorized representative.75 The Assessment Division of the Revenue
Regional Office or the Commissioner or his or her authorized representative is responsible for the
"appropriate review and issuance of a deficiency tax assessment, if warranted."76

If, after the review conducted, there exists sufficient basis to assess the taxpayer with deficiency taxes,
the officer 'shall issue a preliminary assessment notice showing in detail the facts, jurisprudence, and
law on which the assessment is based.77 The taxpayer is given 15 days from receipt of the pre-
assessment notice to respond.78 If the taxpayer fails to respond, he or she will be considered in default,
and a formal letter of demand and assessment notice will be issued.79
The formal letter of demand and assessment notice shall state the facts, jurisprudence, and law on
which the assessment was based; otherwise, these shall be void.80 The taxpayer or the authorized
representative may administratively protest the formal letter of demand and assessment notice within
30 days from receipt of the notice.81

II

The word "shall" in Section 228 of the National Internal Revenue Code and Revenue Regulations No. 12-
99 means the act of informing the taxpayer of both the legal and factual bases of the assessment is
mandatory.82 The law requires that the bases be reflected in the formal letter of demand and
assessment notice.83 This cannot be presumed.84 Otherwise, the express mandate of Section 228 and
Revenue Regulations No. 12-99 would be nugatory.85 The requirement enables the taxpayer to make an
effective protest or appeal of the assessment or decision.86

The rationale behind the requirement that taxpayers should be informed of the facts and the law on
which the assessments are based conforms with the constitutional mandate that no person shall be
deprived of his or her property without due process of law.87 Between the power of the State to tax
and an individual's right to due process, the scale favors the right of the taxpayer to due process.88

The purpose of the written notice requirement is to aid the taxpayer in making a reasonable protest, if
necessary.89Merely notifying the taxpayer of his or her tax liabilities without details or particulars is not
enough.90

Commissioner of Internal Revenue v. United Salvage and Towage (Phils.), Inc.91 held that a final
assessment notice that only contained a table of taxes with no other details was insufficient:

In the present case, a mere perusal of the [Final Assessment Notice] for the deficiency EWT for taxable
year 1994 will show that other than a tabulation of the alleged deficiency taxes due, no further detail
regarding the assessment was provided by petitioner. Only the resulting interest, surcharge and penalty
were anchored with legal basis. Petitioner should have at least attached a detailed notice of discrepancy
or stated an explanation why the amount of P48,461.76 is collectible against respondent and how the
same was arrived at.92

Any deficiency to the mandated content of the assessment or its process will not be
tolerated.93 In Commissioner of Internal Revenue v. Enron,94 an advice of tax deficiency from the
Commissioner of Internal Revenue to an employee of Enron, including the preliminary five (5)-day letter,
were not considered valid substitutes for the mandatory written notice of the legal and factual basis of
the assessment.95 The required issuance of deficiency tax assessment notice to the taxpayer is different
from the required contents of the notice.96 Thus:

The law requires that the legal and factual bases of the assessment be stated in the formal letter of
demand and assessment notice.1âwphi1 Thus, such cannot be presumed. Otherwise, the express
provisions of Article 228 of the [National Internal Revenue Code] and [Revenue Regulations] No. 12-99
would be rendered nugatory. The alleged "factual bases" in the advice, preliminary letter and "audit
working papers" did not suffice. There was no going around the mandate of the law that the legal and
factual bases of the assessment be stated in writing in the formal letter of demand accompanying the
assessment notice.97 (Emphasis supplied)

However, the mandate of giving the taxpayer a notice of the facts and laws on which the assessments
are based should not be mechanically applied.98 To emphasize, the purpose of this requirement is to
sufficiently inform the taxpayer of the bases for the assessment to enable him or her to make an
intelligent protest.99

In Samar-I Electric Cooperative v. Commissioner of Internal Revenue,100 substantial compliance with


Section 228 of the National Internal Revenue Code is allowed, provided that the taxpayer would be later
apprised in writing of the factual and legal bases of the assessment to enable him or her to prepare for
an effective protest.101 Thus:

Although the [Final Assessment Notice] and demand letter issued to petitioner were not accompanied
by a written explanation of the legal and factual bases of the deficiency taxes assessed against the
petitioner, the records showed that respondent in its letter dated April 10, 2003 responded to
petitioner's October 14, 2002 letter-protest, explaining at length the factual and legal bases of the
deficiency tax assessments and denying the protest.

Considering the foregoing exchange of correspondence and documents between the parties, we find
that the requirement of Section 228 was substantially complied with. Respondent had fully informed
petitioner in writing of the factual and legal bases of the deficiency taxes assessment, which enabled the
latter to file an "effective" protest, much unlike the taxpayer's situation in Enron. Petitioner's right to
due process was thus not violated.102

A final assessment notice provides for the amount of tax due with a demand for payment.103 This is to
determine the amount of tax due to a taxpayer.104 However, due process requires that taxpayers be
informed in writing of the facts and law on which the assessment is based in order to aid the taxpayer in
making a reasonable protest.105 To immediately ensue with tax collection without initially
substantiating a valid assessment contravenes the principle in administrative investigations "that
taxpayers should be able to present their case and adduce supporting evidence."106

Respondent filed its income tax return in 1995.107 Almost eight (8) years passed before the disputed
final assessment notice was issued. Respondent pleaded prescription as its defense when it filed a
protest to the Final Assessment Notice. Petitioner claimed fraud assessment to justify the belated
assessment made on respondent.108If fraud was indeed present, the period of assessment should be
within 10 years.109 It is incumbent upon petitioner to clearly state the allegations of fraud committed
by respondent to serve the purpose of an assessment notice to aid respondent in filing an effective
protest.

III
The prescriptive period in making an assessment depends upon whether a tax return was filed or
whether the tax return filed was either false or fraudulent.1âwphi1 When a tax return that is neither
false nor fraudulent has been filed, the Bureau of Internal Revenue may assess within three (3) years,
reckoned from the date of actual filing or from the last day prescribed by law for filing.110 However, in
case of a false or fraudulent return with intent to evade tax, Section 222(a) provides:

Section 222. Exceptions as to Period of Limitation of Assessment and Collection of Taxes. –

(a) In the case of a false or fraudulent return with intent to evade tax or of failure to file a return, the tax
may be assessed, or a proceeding in court for the collection of such tax may be filed without
assessment, at any time within ten (10) years after the discovery of the falsity, fraud or omission:
Provided, That in a fraud assessment which has become final and executory, the fact of fraud shall be
judicially taken cognizance of in the civil or criminal action for the collection thereof. (Emphasis
supplied)

In Aznar v. Court of Tax Appeals,111 this Court interpreted Section 332112 (now Section 222[a] of the
National Internal Revenue Code) by dividing it in three (3) different cases: first, in case of false return;
second, in case of a fraudulent return with intent to evade; and third, in case of failure to file a
return.113 Thus:

Our stand that the law should be interpreted to mean a separation of the three different situations of
false return, fraudulent return with intent to evade tax and failure to file a return is strengthened
immeasurably by the last portion of the provision which aggregates the situations into three different
classes, namely "falsity'', "fraud" and "omission."114

This Court held that there is a difference between "false return" and a "fraudulent return."115 A false
return simply involves a "deviation from the truth, whether intentional or not" while a fraudulent return
"implies intentional or deceitful entry with intent to evade the taxes due."116

Fraud is a question of fact that should be alleged and duly proven.117 "The willful neglect to file the
required tax return or the fraudulent intent to evade the payment of taxes, considering that the same is
accompanied by legal consequences, cannot be presumed."118 Fraud entails corresponding sanctions
under the tax law. Therefore, it is indispensable for the Commissioner of Internal Revenue to include the
basis for its allegations of fraud in the assessment notice.

During the proceedings in the Court of Tax Appeals First Division, respondent presented its President,
Domingo C. Juan Jr. (Juan, Jr.), as witness.119 Juan, Jr. testified that respondent was, in its pre-operating
stage in 1995.120During that period, respondent "imported equipment and distributed them for market
testing in the Philippines without earning any profit."121 He also confirmed that the Final Assessment
Notice and its attachments failed to substantiate the Commissioner's allegations of fraud against
respondent, thus:

More than three (3) years from the time petitioner filed its 1995 annual income tax return on April 11,
1996, respondent issued to petitioner a [Final Assessment Notice] dated March 17, 2004 for the year
1995, pursuant to the Letter of Authority No. 00002953 dated May 13, 2002. The attached Details of
discrepancy containing the assessment for income tax (IT), value-added tax (VAT) and documentary
stamp tax (DST) as well as the Audit Result/ Assessment Notice do not impute fraud on the part of
petitioner. Moreover, it was obtained on information and documents illegally obtained by a [Bureau of
Internal Revenue] informant from petitioner's accountant Elnora Carpio in 1996.122 (Emphasis supplied)

Petitioner did not refute respondent's allegations. For its defense, it presented Socrates Regala (Regala),
the Group Supervisor of the team, who examined respondent's tax liabilities.123 Regala confirmed that
the investigation was prompted by a tip from an informant who provided them with respondent's list of
sales.124 He admitted125 that the gathered information did not show that respondent deliberately
failed to reflect its true income in 1995.126

IV

The issuance of a valid formal assessment is a substantive prerequisite for collection of


taxes.127 Neither the National Internal Revenue Code nor the revenue regulations provide for a
"specific definition or form of an assessment." However, the National Internal Revenue Code defines its
explicit functions and effects."128 An assessment does not only include a computation of tax liabilities;
it also includes a demand for payment within a period prescribed.129 Its main purpose is to determine
the amount that a taxpayer is liable to pay.130

A pre-assessment notice "do[es] not bear the gravity of a formal assessment notice."131 A pre-
assessment notice merely gives a tip regarding the Bureau of Internal Revenue's findings against a
taxpayer for an informal conference or a clarificatory meeting.132

A final assessment is a notice "to the effect that the amount therein stated is due as tax and a demand
for payment thereof."133 This demand for payment signals the time "when penalties and interests
begin to accrue against the taxpayer and enabling the latter to determine his remedies[.]"134 Thus, it
must be "sent to and received by the taxpayer, and must demand payment of the taxes described
therein within a specific period."135

The disputed Final Assessment Notice is not a valid assessment.

First, it lacks the definite amount of tax liability for which respondent is accountable. It does not purport
to be a demand for payment of tax due, which a final assessment notice should supposedly be. An
assessment, in the context of the National Internal Revenue Code, is a "written notice and demand
made by the [Bureau of Internal Revenue] on the taxpayer for the settlement of a due tax liability that is
there: definitely set and fixed."136 Although the disputed notice provides for the computations of
respondent's tax liability, the amount remains indefinite. It only provides that the tax due is still subject
to modification, depending on the date of payment. Thus:

The complete details covering the aforementioned discrepancies established during the investigation of
this case are shown in the accompanying Annex 1 of this Notice. The 50% surcharge and 20% interest
have been imposed pursuant to Sections 248 and 249 (B) of the [National Internal Revenue Code], as
amended. Please note, however, that the interest and the total amount due will have to be adjusted if
prior or beyond April 15, 2004.137 (Emphasis Supplied)

Second, there are no due dates in the Final Assessment Notice. This negates petitioner's demand for
payment.138Petitioner's contention that April 15, 2004 should be regarded as the actual due date
cannot be accepted. The last paragraph of the Final Assessment Notice states that the due dates for
payment were supposedly reflected in the attached assessment:

In view thereof, you are requested to pay your aforesaid deficiency internal revenue tax liabilities
through the duly authorized agent bank in which you are enrolled within the time shown in the enclosed
assessment notice.139 (Emphasis in the original)

However, based on the findings of the Court of Tax Appeals First Division, the enclosed assessment
pertained to remained unaccomplished.140

Contrary to petitioner's view, April 15, 2004 was the reckoning date of accrual of penalties and
surcharges and not the due date for payment of tax liabilities.1avvphi1 The total amount depended
upon when respondent decides to pay. The notice, therefore, did not contain a definite and actual
demand to pay.

Compliance with Section 228 of the National Internal Revenue Code is a substantative
requirement.141 It is not a mere formality.142 Providing the taxpayer with the factual and legal bases
for the assessment is crucial before proceeding with tax collection. Tax collection should be premised on
a valid assessment, which would allow the taxpayer to present his or her case and produce evidence for
substantiation.143

The Court of Tax Appeals did not err in cancelling the Final Assessment Notice as well as the Audit
Result/Assessment Notice issued by petitioner to respondent for the year 1995 covering the "alleged
deficiency income tax, value-added tax and documentary stamp tax amounting to ₱10,647,529.69,
inclusive of surcharges and interest"144 for lack of due process. Thus, the Warrant of Distraint and/or
Levy is void since an invalid assessment bears no valid effect.145

Taxes are the lifeblood of government and should be collected without hindrance.146 However, the
collection of taxes should be exercised "reasonably and in accordance with the prescribed
procedure."147

The essential nature of taxes for the existence of the State grants government with vast remedies to
ensure its collection. However, taxpayers are guaranteed their fundamental right to due process of law,
as articulated in various ways in the process of tax assessment. After all, the State's purpose is to ensure
the well-being of its citizens, not simply to deprive them of their fundamental rights.

WHEREFORE, the Petition is DENIED. The Decision of the Court of Tax Appeals En Banc dated July 14,
2014 and Resolution dated December 16, 2014 in CTA EB Case No. 970 (CTA Case No. 7160) are
hereby AFFIRMED.
SO ORDERED.

MARVIC M.V.F. LEONEN


Associate Justice

WE CONCUR:

ANTONIO T. CARPIO
Associate Justice
Chairperson

ARTURO D. BRION MARIANO C. DEL CASTILLO


Associate Justice Associate Justice

On official leave
JOSE CATRAL MENDOZA*
Associate Justice

ATTESTATION

I attest that the conclusions in the above Decision had been reached in consultation before the case was
assigned to the writer of the opinion of the Court’s Division.

ANTONIO T. CARPIO
Associate Justice
Chairperson, Second Division

CERTIFICATION

Pursuant to the Section 13, Article VIII of the Constitution and the Division Chairperson’s Attestation, I
certify that the conclusions in the above Decision had been reached in consultation before the case was
assigned to the writer of the opinion of the Court’s Division.

MARIA LOURDES P.A. SERENO


Chief Justice

Footnotes

* On official leave.

1 The Petition was filed under Rule 45 of the Rules of Court.


2 Rollo, pp. 32-49. The Decision was penned by Associate Justice Juanito Castafieda, Jr. and concurred in
by Associate Justices Roman G. Del Rosario, Lovell R. Bautista, Erlinda P. Uy, Caesar A. Casanova,
Esperanza R. Fabon-Victorino, Cielito N. Mindaro-Grulla, Amelia R. Cotangco-Manalastas, and Ma. Belen
M. Ringpis-Liban.

3 Id. at 53-57. The Resolution was penned by Associate Justice Juanito Castaneda, Jr. and concurred in
by Associate Justices Roman G. Del Rosario, Lovell R. Bautista, Erlinda P. Uy, Caesar A. Casanova,
Esperanza R. Fabon-Victorino, Cielito N. Mindaro-Grulla, Amelia R. Cotangco-Manalastas, and Ma. Belen
M. Ringpis-Liban.

4 Id. at 48, Court of Tax Appeals En Banc Decision.

5 Id. at 33.

6 Id.

7 Id. at 34.

8 Id. at 36.

9 Id. at 34.

10 Id. at 12-13, Petition for Review. The Annex referred to in the Final Assessment Notice was not
attached to the records of the case. However, based on the testimony of Fitness' President, Domingo C.
Juan, "[t]he attached details of discrepancy containing the assessment for income tax (IT), value-added
tax (VAT) and documentary stamp tax (DST) as well as, the Audit Result/ Assessment Notices do not
impute fraud on the part of petitioner" (Id. at 37, Court of Tax Appeals En Banc Decision).

11 Id.

12 Id.

13 Id.

14 Id.

15 Id.

16 TAX CODE, sec. 222(a) provides:

Section 222. Exceptions as to Period of Limitation of Assessment and Collection of Taxes.

(a) In the case of a false or fraudulent return with intent to evade tax or of failure to file a return, the tax
may be assessed, or a proceeding in court for the collection of such tax may be filed without
assessment, at any time within ten (10) years after the discovery of the falsity, fraud or
omission: Provided, That in a fraud assessment which has become final and executory, the fact of fraud
shall be judicially taken cognizance of in the civil or criminal action for the collection thereof.
17 Rollo, p. 35, Court of Tax Appeals En Banc Decision.

18 Id.

19 Id.

20 Id.

21 Id.

22 Id.

23 Id. at 39.

24 Id. at 35.

25 Id. at 38.

26 Id. at 39.

27 Id.

28 Id. at 67, Petition for Review of the Commissioner of Internal Revenue before the Court of Tax
Appeals En Banc.

29 Id. at 40, Court of Tax Appeals En Banc Decision.

30 TAX CODE, sec. 228 provides:

Section 228. Protesting of Assessment. - When the Commissioner or his duly authorized representative
finds that proper taxes should be assessed, he shall first notify the taxpayer of his findings: Provided,
however, That a pre assessment notice shall, not be required in the following cases:

(a) When the finding for any deficiency tax is the result of mathematical error in the computation of the
tax as appearing on the face of the return; or

(b) When a discrepancy has been determined between the tax withheld and the amount actually
remitted by the withholding agent; or

(c) When a taxpayer who opted to claim a refund or tax credit of excess creditable withholding tax for a
taxable period was determined to have carried over and automatically applied the same amount
claimed against the estimated tax liabilities for the taxable quarter or quarters of the succeeding taxable
year; or

(d) When the excise tax due on excisable articles has not been paid; or
(e) When an article locally purchased or imported by an exempt person, such as, but not limited to,
vehicles, capital equipment, machineries and spare parts, has been sold, traded or transferred to non-
exempt persons.

The taxpayers shall be informed in writing of the law and the facts on which the assessment is made;
otherwise, the assessment shall be void.

Within a period to be prescribed by implementing rules and regulations, the taxpayer shall be required
to respond to said notice. If the taxpayer fails to respond, the' Commissioner or his duly authorized
representative shall issue an assessment based on his findings.

Such assessment may be protested administratively by filing a request for reconsideration or


reinvestigation within thirty (30) days from receipt of the assessment in such form and manner as may
be prescribed by implementing rules and regulations. Within sixty (60) days from filing of the protest, all
relevant supporting documents shall have been submitted; otherwise, the assessment shall become
final.

If the protest is denied in whole or in part, or is not acted upon, within one hundred eighty (180) days
from submission of documents, the taxpayer adversely affected by the decision or inaction may appeal
to the Court of Tax Appeals within thirty (30) days from receipt of the said decision, or from the lapse of
the one hundred eighty (180)-day period; otherwise, the decision shall become final, executory and
demandable.

31 Rollo, pp. 32-33, Court of Tax Appeals En Banc Decision.

32 Id. at 40.

33 Id.

34 Id. at 41.

35 Id.

36 Id.

37 Id.

38 Id. at 48.

39 Id. at 53-57.

40 Implementing the Provisions of the National Internal Revenue Code of 1997 Governing the Rules on
Assessment of National Internal Revenue Taxes, Civil Penalties and Interest and the Extra-Judicial
Settlement of a Taxpayer's Criminal Violation of the Code Through Payment of a Suggested Compromise
Penalty (1999).

BIR Revenue Reg. No. 12-99, sec. 3.1.4 provides:


SECTION 3. Due Process Requirement in the Issuance of a Deficiency Tax Assessment

....

3.1.4 Formal Letter of Demand and Assessment Notice. - The formal letter of demand and assessment
notice shall be issued by the Commissioner or his duly authorized representative. The letter of demand
calling for payment of the taxpayer's deficiency tax or taxes shall state the facts, the law, rules and
regulations, or jurisprudence on which the assessment is based, otherwise, the formal letter of demand
and assessment notice shall be void' (Emphasis supplied).

41 Rollo, p. 16, Petition for Review.

42 Id. at 18.

43 Id.

44 Id. at 20.

45 Id. at 13.

46 Id. at 87.

47 Id. at 90-101.

48 Id. at 91.

49 Id.

50 Id.

51 Id.

52 Id.

53 Id. at 92.

54 Id. at 97.

55 SMI-ED Phil. Technology, Inc. v. Commissioner of Internal Revenue, G.R. No. 175410, November 12,
2014 <http://sc.judiciary. gov.
ph/pdf/web/viewer.html?file=/jurisprudence/2014/november2014/175410.pdf> 5 [Per J. Leonen,
Second Division] .

56 Id.

57 TAX CODE, sec. 56(A) provides:

Section 56. Payment and Assessment of Income Tax for Individuals and Corporations.
A) Payment of Tax. -

(1) In General. - The total amount of tax imposed by this Title shall be paid by the person subject thereto
at the time the return is filed. In the case of tramp vessels, the shipping agents and/or the husbanding
agents, and in their absence, the captains thereof are required to file the return herein provided and pay
the tax due thereon before their departure. Upon failure of the said agents or captains to file the return
and pay the tax, the Bureau of Customs is hereby authorized to hold the vessel and prevent its
departure until proof of payment of the tax is presented or a sufficient bond is filed to answer for the
tax due.

(2) Installment Payment. - When the tax due is in excess of Two thousand pesos (₱2,000), the taxpayer
other than a corporation may elect to pay the tax in two (2) equal installments in which case, the first
installment shall be paid at the time the return is filed and the second installment, on or before July 15
following the close of the calendar year. If any installment is not paid on or before the date fixed for its
payment, the whole amount of the tax unpaid becomes due and payable, together with the delinquency
penalties.

(3) Payment of Capital Gains Tax. -The total amount of tax imposed and prescribed under Sections 24(C),
24(D), 27(E)(2), 28(A)(8)(c) and 28(B)(5)(c) shall be paid on the date the return prescribed therefor is
filed by the person liable thereto: Provided, That if the seller submits proof of his intention to avail
himself of the benefit of exemption of capital gains under existing special laws, no such payments shall
be required: Provided, further, That in case of failure to qualify for exemption under such special laws
and implementing rules and regulations, the tax due on the gains realized from the original transaction
shall immediately become due and payable, and subject to the penalties prescribed under applicable
provisions of this Code: Provided, finally, That if the seller, having paid the tax, submits such proof of
intent within six (6) months from the registration of the document transferring the real property, he
shall be entitled to a refund of such tax upon verification of his compliance with the requirements for
such exemption.

In case the taxpayer elects and is qualified to report the gain by installments under Section 49 of this
Code, the tax due from each installment payment shall be paid within thirty (30) days from the receipt
of such payments.

No registration of any document transferring real property shall be effected by the Register of Deeds
unless the Commissioner or his duly authorized representative has certified that such transfer has been
reported, and the tax herein imposed, if any, has been paid.

58 SMI-ED Phil. Technology, Inc. v. Commissioner of Internal Revenue, G.R. No. 175410, November 12,
2014
<http://sc.judiciary.gov.ph/pdf/web/viewer.html?file=/jurisprudence/2014/november2014/175410.pdt
> 8 [Per J. Leonen, Second Division]

59 Id.
60 Id.

61 TAX CODE, sec. 6(A).

62 TAX CODE, sec. 6(A).

63 TAX CODE, sec. 5(A).

64 Commissioner of Internal Revenue v. Liquigaz Philippines Corp., G.R. Nos. 215534 & 215557, April 18,
2016 <sc.judiciary.gov.ph/pdf/web/viewer.html?file=/jurisprudence/20l6/april2016/215534.pdt> 7 [Per
J. Mendoza, Second Division].

65 Id.

66 On November 28, 2013, Revenue Regulations No. 18-2013 was enacted amending Certain Sections of
Revenue Regulations No. 12-99 relative to the Due Process Requirement in the Issuance of a Deficiency
Tax Assessment. The scope of the law provides that under the provisions of Section 244, in relation to
Section 245 of the National Internal Revenue Code, as amended, these Regulations are promulgated to
amend provisions of Revenue Regulations No. 12-99.

BIR Revenue Reg. No. 18-2013, sec. 2 provides:

Section 2. Amendment. - Section 3 of RR 12-99 is hereby amended by deleting Section 3.1.1 thereof
which provides for the preparation of a Notice of Informal Conference, thereby renumbering other
provisions thereof, and prescribing other provisions for the assessment of tax liabilities.

67 BIR Revenue Reg. No. 12-99, sec. 3.1.1.

68 BIR Revenue Reg. No. 12-99, sec. 3.1.1.

69 BIR Revenue Reg. No. 12-99, sec. 3.1.1 provides that either the Revenue District Office or the Special
Investigation Division, as the case may be (in the case of Revenue Regional Offices) or the Chief of
Division concerned (in the case of the BIR National Office) may inform the taxpayer of his or her
discrepancies.

70 BIR Revenue Reg. No. 12-99, sec. 3.1.1.

71 BIR Revenue Reg. No. 12-99, sec. 3.1.1.

72 BIR Revenue Reg. No. 12-99, sec. 3.1.l.

73 BIR Revenue Reg. No. 12-99, sec. 3.1.l.

74 Revenue Reg. No. 12-99, sec. 3.1.1 provides that in case of default, the "Revenue District Officer or
the Chief of the Special Investigation Division of the Revenue Regional Office, or the Chief of Division in
the National Office" shall endorse the case to the Assessment Division.

75 BIR Revenue Reg. No. 12-99, sec. 3.1.1.


76 BIR Revenue Reg. No. 12-99, sec. 3.1.l.

77 BIR Revenue Reg. No. 12-99, sec. 3.1.2.

78 BIR Revenue Reg. No. 12-99, sec. 3.1.1.

79 BIR Revenue Reg. No. 12-99, sec. 3.1.1.

80 BIR Revenue Reg. No. 12-99, sec. 3.1.4.

81 BIR Revenue Reg. No. 12-99, sec. 3.1.4.

82 Commissioner of Internal Revenue v. Liquigaz Philippines Corp., G.R. Nos. 215534 & 215557, April 18,
2016 <http://sc.judiciary.gov.
ph/pdf/web/viewer.htrnl?file=/jurisprudence/2016/april2016/215534.pdf> [Per J. Mendoza, Second
Division]; Commissioner of Internal Revenue v. Enron Subic Power Corp., 596 Phil. 229 (2009) [Per J.
Corona, First Division]; Commissioner of Internal Revenue v. United Salvage and Towage (Phils), Inc., 738
Phil. 335 (2014) [Per J. Peralta, Third Division].

83 Commissioner of Internal Revenue v. Enron Subic Power Corp., 596 Phil. 229, 235 (2009) [Per J.
Corona, First Division].

84 Id.

85 Id.

86 Commissioner of Internal Revenue v. Liquigaz Philippines Corp., G.R. Nos. 215534 & 215557, April 18,
2016 <http://sc.judiciary.gov.ph/pdf/web/viewer.html?file=/jurisprudence/20l6/april2016/215534.pdf>
14 [Per J. Mendoza, Second Division].

87 Id.

88 Commissioner of Internal Revenue v. Metro Star Superama, Inc., 652 Phil. 172,187 (2010) [Per J.
Mendoza, Second Division].

89 Commissioner of Internal Revenue v. Liquigaz Philippines Corp., G.R. Nos. 215534 & 215557, April 18,
2016 <http://sc.judiciary.gov. ph/pdf/web/viewer.html?file=/jurisprudence/20l6/april2016/215534.pdf>
12 [Per J. Mendoza, Second Division].

90 Id., citing Commissioner of Internal Revenue v. Reyes, 516 Phil. 176, 186-190 (2006) [Per C.J.
Panganiban, First Division].

91 738 Phil. 335 (2014) [Per J. Peralta, Third Division]

92 Id. at 349-350.

93 Commissioner of Internal Revenue v. Liquigaz Philippines Corp., G.R. Nos. 215534 & 215557, April 18,
2016
<http://sc.judiciary.gov.ph/pdf/web/viewer.html?file=/jurisprudence/2016/april2016/215534.pdt> [Per
J. Mendoza, Second Division], citing Commissioner of Internal Revenue v. United Salvage and Towage
(Phils), Inc., 738 Phil. 335 (2014) [Per J. Peralta, Third Division], in turn citing Commissioner of Internal
Revenue v. Enron Subic Power Corp., 596 Phil. 229 (2009) [Per J. Corona, First Division].

94 596 Phil. 229 (2009) [Per J. Corona, First Division].

95 Id. at 235-236.

96 Id. at 236.

97 Id.

98 Commissioner of Internal Revenue v. Liquigaz Philippines Corp., G.R. Nos. 215534 & 215557, April 18,
2016
<http://sc.judiciary.gov.ph/pdf/web/viewer.html?file=/jurisprudence/2016/april2016/215534.pdt> 14-
15 [Per J. Mendoza, Second Division].

99 Id.

100 G.R. No. 193100, December 10, 2014


<http://sc.judiciary.gov.ph/pdf/web/viewer.html?file=/jurisprudence/20l4/december2014/193100.pdf>
[Per J. Villarama, Jr., Third Division].

101 Id. at 12.

102 Id.

103 Commissioner of Internal Revenue v. Menguito, 587 Phil. 234, 256 (2008) [Per J. Austria-Martinez,
Third Division].

104 Tupaz v. Ulep, 374 Phil. 474, 484 (1999) [Per J. Pardo, First Division].

105 Commissioner of Internal Revenue v. Liquigaz Philippines Corp., G.R. Nos. 215534 & 215557, April
18, 2016 <http://sc.judiciary.gov.
ph/pdf/web/viewer.html?file=/jurisprudence/20l6/april2016/215534.pdf> 15 [Per J. Mendoza, Second
Division],

106 Commissioner of Internal Revenue v. Reyes, 516 Phil. 176, 190 (2006) [Per C.J. Panganiban, First
Division]

107 Rollo, p. 33.

108 Id. at 34.

109 TAX CODE, sec. 222(a).

110 TAX CODE, sec. 203 provides:


Section 203. Period of Limitation Upon Assessment and Collection. - Except as provided in Section 222,
internal revenue taxes shall be assessed within three (3) years after the last day prescribed by law for
the filing of the return, and no proceeding in court without assessment for the collection of such taxes
shall be begun after the expiration of such period: Provided, That in a case where a return is filed
beyond the period prescribed by law, the three (3)-year period shall be counted from the day the return
was filed. For purposes of this Section, a return filed before the last day prescribed by law for the filing
thereof shall be considered as filed on such last day.

111 157 Phil. 510 (1974) [Per J. Esguerra, First Division].

112 TAX CODE, sec. 222(a) provides:

(a) In the case of a false or fraudulent return with intent to evade tax or of a failure to file a return, the
tax may be assessed, or a proceeding in court for the collection of such tax may be begun without
assessment, at any time within ten years after the discovery of the falsity, fraud or omission.

125 Id. Regala admitted that "[i]n their memorandum report, they recommended the filing of a civil case
for the collection of petitioner's tax liabilities and a criminal case, for its failure to declare in its ITR for
the year 1995 the income derived from its cited sales. Thus, the BIR's filing of a criminal case against
petitioner with the Department of Justice (DOJ). The witness confirmed that the gathered
information did not indicate that petitioner's failure to state in its ITR its income and sales for the year
1995 was deliberate. The instant case was precipitated by the issuance of the Letter of Authority on May
13, 2002." (Emphasis supplied)

DECISION

CALLEJO, SR., J.:

Before us is a petition for review of the Decision[1] of the Court of Appeals (CA) which reversed the
Decision[2] of the Court of Tax Appeals (CTA) in CTA Case No. 5126, upholding the deficiency income
and sales tax assessments against respondent Hantex Trading Co., Inc.

The Antecedents

The respondent is a corporation duly organized and existing under the laws of the Philippines. Being
engaged in the sale of plastic products, it imports synthetic resin and other chemicals for the
manufacture of its products. For this purpose, it is required to file an Import Entry and Internal Revenue
Declaration (Consumption Entry) with the Bureau of Customs under Section 1301 of the Tariff and
Customs Code.

Sometime in October 1989, Lt. Vicente Amoto, Acting Chief of Counter-Intelligence Division of the
Economic Intelligence and Investigation Bureau (EIIB), received confidential information that the
respondent had imported synthetic resin amounting to P115,599,018.00 but only
declared P45,538,694.57.[3] According to the informer, based on photocopies of 77 Consumption
Entries furnished by another informer, the 1987 importations of the respondent were understated in its
accounting records.[4] Amoto submitted a report to the EIIB Commissioner recommending that an
inventory audit of the respondent be conducted by the Internal Inquiry and Prosecution Office (IIPO) of
the EIIB.[5]

Acting on the said report, Jose T. Almonte, then Commissioner of the EIIB, issued Mission Order No. 398-
89[6] dated November 14, 1989 for the audit and investigation of the importations of Hantex for 1987.
The IIPO issued subpoena duces tecum and ad testificandum for the president and general manager of
the respondent to appear in a hearing and bring the following:

1. Books of Accounts for the year 1987;

2. Record of Importations of Synthetic Resin and Calcium Carbonate for the year 1987;

3. Income tax returns & attachments for 1987; and

4. Record of tax payments.[7]

However, the respondents president and general manager refused to comply with the subpoena,
contending that its books of accounts and records of importation of synthetic resin and calcium
bicarbonate had been investigated repeatedly by the Bureau of Internal Revenue (BIR) on prior
occasions.[8] The IIPO explained that despite such previous investigations, the EIIB was still authorized
to conduct an investigation pursuant to Section 26-A of Executive Order No. 127. Still, the respondent
refused to comply with the subpoena issued by the IIPO. The latter forthwith secured certified copies of
the Profit and Loss Statements for 1987 filed by the respondent with the Securities and Exchange
Commission (SEC).[9] However, the IIPO failed to secure certified copies of the respondents 1987
Consumption Entries from the Bureau of Customs since, according to the custodian thereof, the original
copies had been eaten by termites.[10]

In a Letter dated June 28, 1990, the IIPO requested the Chief of the Collection Division, Manila
International Container Port, and the Acting Chief of the Collection Division, Port of Manila, to
authenticate the machine copies of the import entries supplied by the informer. However, Chief of the
Collection Division Merlita D. Tomas could not do so because the Collection Division did not have the
original copies of the entries. Instead, she wrote the IIPO that, as gleaned from the records, the
following entries had been duly processed and released after the payment of duties and taxes:

IMPORTER HANTEX TRADING CO., INC. SERIES OF 1987

ENTRY NO. DATE RELEASED ENTRY NO. DATE RELEASED

03058-87 1-30-87 50265-87 12-09-87

09120-87 3-20-87 46427-87 11-27-87

18089-87 5-21-87 30764-87 8-21-87

19439-87 6-2-87 30833-87 8-20-87


19441-87 6-3-87 34690-87 9-16-87

11667-87 4-15-87 34722-87 9-11-87

23294-87 7-7-87 43234-87 11-2-87

45478-87 11-16-87 44850-87 11-16-87

45691-87 12-2-87 44851-87 11-16-87

25464-87 7-16-87 46461-87 11-19-87

26483-87 7-23-87 46467-87 11-18-87

29950-87 8-11-87 48091-87 11-27-87[11]

Acting Chief of the Collection Division of the Bureau of Customs Augusto S. Danganan could not
authenticate the machine copies of the import entries as well, since the original copies of the said
entries filed with the Bureau of Customs had apparently been eaten by termites. However, he issued a
certification that the following enumerated entries were filed by the respondent which were processed
and released from the Port of Manila after payment of duties and taxes, to wit:

Hantex Trading Co., Inc.

Entry No. Date Released Entry No. Date Released

03903 1-29-87 22869 4-8-87

04414 1-20-87 19441 3-31-87

10683 2-17-87 24189 4-21-87

12611 2-24-87 26431 4-20-87

12989 2-26-87 45478 7-3-87

17050 3-13-87 26796 4-23-87

17169 3-13-87 28827 4-30-87

18089 3-16-87 31617 5-14-87

19439 4-1-87 39068 6-5-87

21189 4-3-87 42581 6-21-87

43451 6-29-87 42793 6-23-87

42795 6-23-87 45477 7-3-87


35582 not received 85830 11-13-87

45691 7-3-87 86650 not received

46187 7-8-87 87647 11-18-87

46427 7-3-87 88829 11-23-87

57669 8-12-87 92293 12-3-87

62471 8-28-87 93292 12-7-87

63187 9-2-87 96357 12-16-87

66859 9-15-87 96822 12-15-87

67890 9-17-87 98823 not received

68115 9-15-87 99428 12-28-87

69974 9-24-87 99429 12-28-87

72213 10-2-87 99441 12-28-87

77688 10-16-87 101406 1-5-87

84253 11-10-87 101407 1-8-87

85534 11-11-87 03118 1-19-87[12]

Bienvenido G. Flores, Chief of the Investigation Division, and Lt. Leo Dionela, Lt. Vicente Amoto and Lt.
Rolando Gatmaitan conducted an investigation. They relied on the certified copies of the respondents
Profit and Loss Statement for 1987 and 1988 on file with the SEC, the machine copies of the
Consumption Entries, Series of 1987, submitted by the informer, as well as excerpts from the entries
certified by Tomas and Danganan.

Based on the documents/records on hand, inclusive of the machine copies of the Consumption Entries,
the EIIB found that for 1987, the respondent had importations totaling P105,716,527.00 (inclusive of
advance sales tax). Compared with the declared sales based on the Profit and Loss Statements filed with
the SEC, the respondent had unreported sales in the amount of P63,032,989.17, and its corresponding
income tax liability was P41,916,937.78, inclusive of penalty charge and interests.

EIIB Commissioner Almonte transmitted the entire docket of the case to the BIR and recommended the
collection of the total tax assessment from the respondent.[13]

On February 12, 1991, Deputy Commissioner Deoferio, Jr. issued a Memorandum to the BIR Assistant
Commissioner for Special Operations Service, directing the latter to prepare a conference letter advising
the respondent of its deficiency taxes.[14]
Meanwhile, as ordered by the Regional Director, Revenue Enforcement Officers Saturnino D. Torres and
Wilson Filamor conducted an investigation on the 1987 importations of the respondent, in the light of
the records elevated by the EIIB to the BIR, inclusive of the photocopies of the Consumption Entries.
They were to ascertain the respondents liability for deficiency sales and income taxes for 1987, if any.
Per Torres and Filamors Report dated March 6, 1991 which was based on the report of the EIIB and the
documents/records appended thereto, there was a prima facie case of fraud against the respondent in
filing its 1987 Consumption Entry reports with the Bureau of Customs. They found that the respondent
had unrecorded importation in the total amount of P70,661,694.00, and that the amount was not
declared in its income tax return for 1987. The District Revenue Officer and the Regional Director of the
BIR concurred with the report.[15]

Based on the said report, the Acting Chief of the Special Investigation Branch wrote the respondent and
invited its representative to a conference at 10:00 a.m. of March 14, 1991 to discuss its deficiency
internal revenue taxes and to present whatever documentary and other evidence to refute the
same.[16] Appended to the letter was a computation of the deficiency income and sales tax due from
the respondent, inclusive of increments:

B. Computations:

1. Cost of Sales Ratio A2/A1 85.492923%

2. Undeclared Sales Imported A3/B1 110,079,491.61

3. Undeclared Gross Profit B2-A3 15,969,316.61

C. Deficiency Taxes Due:

1. Deficiency Income Tax B3 x 35% 5,589,261.00

50% Surcharge C1 x 50% 2,794,630.50

Interest to 2/28/91 C1 x 57.5% 3,213,825.08

Total 11,597,825.58

2. Deficiency Sales Tax

at 10% 7,290,082.72

at 20% 10,493,312.31

Total Due 17,783,395.03

Less: Advanced Sales Taxes Paid 11,636,352.00

Deficiency Sales Tax 6,147,043.03

50% Surcharge C2 x 50% 3,073,521.52


Interest to 2/28/91 5,532,338.73

Total 14,752,903.28[17]

===========

The invitation was reiterated in a Letter dated March 15, 1991. In his Reply dated March 15, 1991,
Mariano O. Chua, the President and General Manager of the respondent, requested that the report of
Torres and Filamor be set aside on the following claim:

[W]e had already been investigated by RDO No. 23 under Letters of Authority Nos. 0322988 RR dated
Oct. 1, 1987, 0393561 RR dated Aug. 17, 1988 and 0347838 RR dated March 2, 1988, and re-investigated
by the Special Investigation Team on Aug. 17, 1988 under Letter of Authority No. 0357464 RR, and the
Intelligence and Investigation Office on Sept. 27, 1988 under Letter of Authority No. 0020188 NA, all for
income and business tax liabilities for 1987. The Economic Intelligence and Investigation Bureau on Nov.
20, 1989, likewise, confronted us on the same information for the same year.

In all of these investigations, save your request for an informal conference, we welcomed them and
proved the contrary of the allegation. Now, with your new inquiry, we think that there will be no end to
the problem.

Madam, we had been subjected to so many investigations and re-investigations for 1987 and nothing
came out except the payment of deficiency taxes as a result of oversight. Tax evasion through
underdeclaration of income had never been proven.[18]

Invoking Section 235[19] of the 1977 National Internal Revenue Code (NIRC), as amended, Chua
requested that the inquiry be set aside.

The petitioner, the Commissioner of Internal Revenue, through Assistant Commissioner for Collection
Jaime M. Maza, sent a Letter dated April 15, 1991 to the respondent demanding payment of its
deficiency income tax of P13,414,226.40 and deficiency sales tax of P14,752,903.25, inclusive of
surcharge and interest.[20] Appended thereto were the Assessment Notices of Tax Deficiency Nos. FAS-
1-87-91-001654 and FAS-4-87-91-001655.[21]

On February 12, 1992, the Chief of the Accounts Receivables/Billing Division of the BIR sent a letter to
the respondent demanding payment of its tax liability due for 1987 within ten (10) days from notice, on
pain of the collection tax due via a warrant of distraint and levy and/or judicial action.[22] The Warrant
of Distraint and/or Levy[23] was actually served on the respondent on January 21, 1992. On September
7, 1992, it wrote the Commissioner of Internal Revenue protesting the assessment on the following
grounds:

I. THAT THE ASSESSMENT HAS NO FACTUAL AS WELL AS LEGAL BASIS, THE FACT THAT NO
INVESTIGATION OF OUR RECORDS WAS EVER MADE BY THE EIIB WHICH RECOMMENDED ITS
ISSUANCE.[24]
II. THAT GRANTING BUT WITHOUT ADMITTING THAT OUR PURCHASES FOR 1987 AMOUNTED
TO P105,716,527.00 AS CLAIMED BY THE EIIB, THE ASSESSMENT OF A DEFICIENCY INCOME TAX IS STILL
DEFECTIVE FOR IT FAILED TO CONSIDER OUR REAL PURCHASES OF P45,538,694.57.[25]

III. THAT THE ASSESSMENT OF A DEFICIENCY SALES TAX IS ALSO BASELESS AND UNFOUNDED
CONSIDERING THAT WE HAVE DUTIFULLY PAID THE SALES TAX DUE FROM OUR BUSINESS.[26]

In view of the impasse, administrative hearings were conducted on the respondents protest to the
assessment. During the hearing of August 20, 1993, the IIPO representative presented the photocopies
of the Consumption and Import Entries and the Certifications issued by Tomas and Danganan of the
Bureau of Customs. The IIPO representative testified that the Bureau of Customs failed to furnish the
EIIB with certified copies of the Consumption and Import Entries; hence, the EIIB relied on the machine
copies from their informer.[27]

The respondent wrote the BIR Commissioner on July 12, 1993 questioning the assessment on the
ground that the EIIB representative failed to present the original, or authenticated, or duly certified
copies of the Consumption and Import Entry Accounts, or excerpts thereof if the original copies were
not readily available; or, if the originals were in the official custody of a public officer, certified copies
thereof as provided for in Section 12, Chapter 3, Book VII, Administrative Procedure, Administrative
Order of 1987. It stated that the only copies of the Consumption Entries submitted to the Hearing
Officer were mere machine copies furnished by an informer of the EIIB. It asserted that the letters of
Tomas and Danganan were unreliable because of the following:

In the said letters, the two collection officers merely submitted a listing of alleged import entry numbers
and dates released of alleged importations by Hantex Trading Co., Inc. of merchandise in 1987, for which
they certified that the corresponding duties and taxes were paid after being processed in their offices. In
said letters, no amounts of the landed costs and advance sales tax and duties were stated, and no
particulars of the duties and taxes paid per import entry document was presented.

The contents of the two letters failed to indicate the particulars of the importations per entry number,
and the said letters do not constitute as evidence of the amounts of importations of Hantex Trading Co.,
Inc. in 1987.[28]

The respondent cited the following findings of the Hearing Officer:

[T]hat the import entry documents do not constitute evidence only indicate that the tax assessments in
question have no factual basis, and must, at this point in time, be withdrawn and cancelled. Any new
findings by the IIPO representative who attended the hearing could not be used as evidence in this
hearing, because all the issues on the tax assessments in question have already been raised by the
herein taxpayer.[29]

The respondent requested anew that the income tax deficiency assessment and the sales tax deficiency
assessment be set aside for lack of factual and legal basis.
The BIR Commissioner[30] wrote the respondent on December 10, 1993, denying its letter-request for
the dismissal of the assessments.[31] The BIR Commissioner admitted, in the said letter, the possibility
that the figures appearing in the photocopies of the Consumption Entries had been tampered with. She
averred, however, that she was not proscribed from relying on other admissible evidence, namely, the
Letters of Torres and Filamor dated August 7 and 22, 1990 on their investigation of the respondents tax
liability. The Commissioner emphasized that her decision was final.[32]

The respondent forthwith filed a petition for review in the CTA of the Commissioners Final Assessment
Letter dated December 10, 1993 on the following grounds:

First. The alleged 1987 deficiency income tax assessment (including increments) and the alleged 1987
deficiency sales tax assessment (including increments) are void ab initio, since under Sections 16(a) and
49(b) of the Tax Code, the Commissioner shall examine a return after it is filed and, thereafter, assess
the correct amount of tax. The following facts obtaining in this case, however, are indicative of the
incorrectness of the tax assessments in question: the deficiency interests imposed in the income and
percentage tax deficiency assessment notices were computed in violation of the provisions of Section
249(b) of the NIRC of 1977, as amended; the percentage tax deficiency was computed on an annual
basis for the year 1987 in accordance with the provision of Section 193, which should have been
computed in accordance with Section 162 of the 1977 NIRC, as amended by Pres. Decree No. 1994 on a
quarterly basis; and the BIR official who signed the deficiency tax assessments was the Assistant
Commissioner for Collection, who had no authority to sign the same under the NIRC.

Second. Even granting arguendo that the deficiency taxes and increments for 1987 against the
respondent were correctly computed in accordance with the provisions of the Tax Code, the facts
indicate that the above-stated assessments were based on alleged documents which are inadmissible in
either administrative or judicial proceedings. Moreover, the alleged bases of the tax computations were
anchored on mere presumptions and not on actual facts. The alleged undeclared purchases for 1987
were based on mere photocopies of alleged import entry documents, not the original ones, and which
had never been duly certified by the public officer charged with the custody of such records in the
Bureau of Customs. According to the respondent, the alleged undeclared sales were computed based on
mere presumptions as to the alleged gross profit contained in its 1987 financial statement. Moreover,
even the alleged financial statement of the respondent was a mere machine copy and not an official
copy of the 1987 income and business tax returns. Finally, the respondent was following the accrual
method of accounting in 1987, yet, the BIR investigator who computed the 1987 income tax deficiency
failed to allow as a deductible item the alleged sales tax deficiency for 1987 as provided for under
Section 30(c) of the NIRC of 1986.[33]

The Commissioner did not adduce in evidence the original or certified true copies of the 1987
Consumption Entries on file with the Commission on Audit. Instead, she offered in evidence as proof of
the contents thereof, the photocopies of the Consumption Entries which the respondent objected to for
being inadmissible in evidence.[34] She also failed to present any witness to prove the correct amount
of tax due from it. Nevertheless, the CTA provisionally admitted the said documents in evidence, subject
to its final evaluation of their relevancy and probative weight to the issues involved.[35]
On December 11, 1997, the CTA rendered a decision, the dispositive portion of which reads:

IN THE LIGHT OF ALL THE FOREGOING, judgment is hereby rendered DENYING the herein petition.
Petitioner is hereby ORDERED TO PAY the respondent Commissioner of Internal Revenue its deficiency
income and sales taxes for the year 1987 in the amounts of P11,182,350.26 and P12,660,382.46,
respectively, plus 20% delinquency interest per annum on both deficiency taxes from April 15, 1991 until
fully paid pursuant to Section 283(c)(3) of the 1987 Tax Code, with costs against the petitioner.

SO ORDERED.[36]

The CTA ruled that the respondent was burdened to prove not only that the assessment was erroneous,
but also to adduce the correct taxes to be paid by it. The CTA declared that the respondent failed to
prove the correct amount of taxes due to the BIR. It also ruled that the respondent was burdened to
adduce in evidence a certification from the Bureau of Customs that the Consumption Entries in question
did not belong to it.

On appeal, the CA granted the petition and reversed the decision of the CTA. The dispositive portion of
the decision reads:

FOREGOING PREMISES CONSIDERED, the Petition for Review is GRANTED and the December 11, 1997
decision of the CTA in CTA Case No. 5162 affirming the 1987 deficiency income and sales tax
assessments and the increments thereof, issued by the BIR is hereby REVERSED. No costs.[37]

The Ruling of the Court of Appeals

The CA held that the income and sales tax deficiency assessments issued by the petitioner were
unlawful and baseless since the copies of the import entries relied upon in computing the deficiency tax
of the respondent were not duly authenticated by the public officer charged with their custody, nor
verified under oath by the EIIB and the BIR investigators.[38] The CA also noted that the public officer
charged with the custody of the import entries was never presented in court to lend credence to the
alleged loss of the originals.[39] The CA pointed out that an import entry is a public document which
falls within the provisions of Section 19, Rule 132 of the Rules of Court, and to be admissible for any
legal purpose, Section 24, Rule 132 of the Rules of Court should apply.[40] Citing the ruling of this Court
in Collector of Internal Revenue v. Benipayo,[41] the CA ruled that the assessments were unlawful
because they were based on hearsay evidence. The CA also ruled that the respondent was deprived of
its right to due process of law.

The CA added that the CTA should not have just brushed aside the legal requisites provided for under
the pertinent provisions of the Rules of Court in the matter of the admissibility of public documents,
considering that substantive rules of evidence should not be disregarded. It also ruled that the
certifications made by the two Customs Collection Chiefs under the guise of supporting the respondents
alleged tax deficiency assessments invoking the best evidence obtainable rule under the Tax Code
should not be permitted to supplant the best evidence rule under Section 7, Rule 130 of the Rules of
Court.
Finally, the CA noted that the tax deficiency assessments were computed without the tax returns. The
CA opined that the use of the tax returns is indispensable in the computation of a tax deficiency; hence,
this essential requirement must be complied with in the preparation and issuance of valid tax deficiency
assessments.[42]

The Present Petition

The Commissioner of Internal Revenue, the petitioner herein, filed the present petition for review under
Rule 45 of the Rules of Court for the reversal of the decision of the CA and for the reinstatement of the
ruling of the CTA.

As gleaned from the pleadings of the parties, the threshold issues for resolution are the following: (a)
whether the petition at bench is proper and complies with Sections 4 and 5, Rule 7 of the Rules of Court;
(b) whether the December 10, 1991 final assessment of the petitioner against the respondent for
deficiency income tax and sales tax for the latters 1987 importation of resins and calcium bicarbonate is
based on competent evidence and the law; and (c) the total amount of deficiency taxes due from the
respondent for 1987, if any.

On the first issue, the respondent points out that the petition raises both questions of facts and law
which cannot be the subject of an appeal by certiorari under Rule 45 of the Rules of Court. The
respondent notes that the petition is defective because the verification and the certification against
forum shopping were not signed by the petitioner herself, but only by the Regional Director of the BIR.
The respondent submits that the petitioner should have filed a motion for reconsideration with the CA
before filing the instant petition for review.[43]

We find and so rule that the petition is sufficient in form. A verification and certification against forum
shopping signed by the Regional Director constitutes sufficient compliance with the requirements of
Sections 4 and 5, Rule 7 of the Rules of Court. Under Section 10 of the NIRC of 1997,[44] the Regional
Director has the power to administer and enforce internal revenue laws, rules and regulations, including
the assessment and collection of all internal revenue taxes, charges and fees. Such power is broad
enough to vest the Revenue Regional Director with the authority to sign the verification and certification
against forum shopping in behalf of the Commissioner of Internal Revenue. There is no other person in a
better position to know the collection cases filed under his jurisdiction than the Revenue Regional
Director.

Moreover, under Revenue Administrative Order No. 5-83,[45] the Regional Director is authorized to sign
all pleadings filed in connection with cases referred to the Revenue Regions by the National Office
which, otherwise, require the signature of the petitioner.

We do not agree with the contention of the respondent that a motion for reconsideration ought to have
been filed before the filing of the instant petition. A motion for reconsideration of the decision of the CA
is not a condition sine qua non for the filing of a petition for review under Rule 45. As we held in Almora
v. Court of Appeals:[46]
Rule 45, Sec. 1 of the Rules of Court, however, distinctly provides that:

A party may appeal by certiorari from a judgment of the Court of Appeals, by filing with the Supreme
Court a petition for certiorari within fifteen (15) days from notice of judgment, or of the denial of his
motion for reconsideration filed in due time. (Emphasis supplied)

The conjunctive or clearly indicates that the 15-day reglementary period for the filing of a petition for
certiorari under Rule 45 commences either from notice of the questioned judgment or from notice of
denial of the appellants motion for reconsideration. A prior motion for reconsideration is not
indispensable for a petition for review on certiorari under Rule 45 to prosper. [47]

While Rule 45 of the Rules of Court provides that only questions of law may be raised by the petitioner
and resolved by the Court, under exceptional circumstances, the Court may take cognizance thereof and
resolve questions of fact. In this case, the findings and conclusion of the CA are inconsistent with those
of the CTA, not to mention those of the Commissioner of Internal Revenue. The issues raised in this case
relate to the propriety and the correctness of the tax assessments made by the petitioner against the
respondent, as well as the propriety of the application of Section 16, paragraph (b) of the 1977 NIRC, as
amended by Pres. Decree Nos. 1705, 1773, 1994 and Executive Order No. 273, in relation to Section 3,
Rule 132 of the Rules of Evidence. There is also an imperative need for the Court to resolve the
threshold factual issues to give justice to the parties, and to determine whether the CA capriciously
ignored, misunderstood or misinterpreted cogent facts and circumstances which, if considered, would
change the outcome of the case.

On the second issue, the petitioner asserts that since the respondent refused to cooperate and show its
1987 books of account and other accounting records, it was proper for her to resort to the best evidence
obtainable the photocopies of the import entries in the Bureau of Customs and the respondents
financial statement filed with the SEC.[48] The petitioner maintains that these import entries were
admissible as secondary evidence under the best evidence obtainable rule, since they were duly
authenticated by the Bureau of Customs officials who processed the documents and released the
cargoes after payment of the duties and taxes due.[49] Further, the petitioner points out that under the
best evidence obtainable rule, the tax return is not important in computing the tax deficiency.[50]

The petitioner avers that the best evidence obtainable rule under Section 16 of the 1977 NIRC, as
amended, legally cannot be equated to the best evidence rule under the Rules of Court; nor can the best
evidence rule, being procedural law, be made strictly operative in the interpretation of the best
evidence obtainable rule which is substantive in character.[51] The petitioner posits that the CTA is not
strictly bound by technical rules of evidence, the reason being that the quantum of evidence required in
the said court is merely substantial evidence.[52]

Finally, the petitioner avers that the respondent has the burden of proof to show the correct
assessments; otherwise, the presumption in favor of the correctness of the assessments made by it
stands.[53] Since the respondent was allowed to explain its side, there was no violation of due
process.[54]
The respondent, for its part, maintains that the resort to the best evidence obtainable method was
illegal. In the first place, the respondent argues, the EIIB agents are not duly authorized to undertake
examination of the taxpayers accounting records for internal revenue tax purposes. Hence, the
respondents failure to accede to their demands to show its books of accounts and other accounting
records cannot justify resort to the use of the best evidence obtainable method.[55] Secondly, when a
taxpayer fails to submit its tax records upon demand by the BIR officer, the remedy is not to assess him
and resort to the best evidence obtainable rule, but to punish the taxpayer according to the provisions
of the Tax Code.[56]

In any case, the respondent argues that the photocopies of import entries cannot be used in making the
assessment because they were not properly authenticated, pursuant to the provisions of Sections
24[57] and 25[58] of Rule 132 of the Rules of Court. It avers that while the CTA is not bound by the
technical rules of evidence, it is bound by substantial rules.[59]The respondent points out that the
petitioner did not even secure a certification of the fact of loss of the original documents from the
custodian of the import entries. It simply relied on the report of the EIIB agents that the import entry
documents were no longer available because they were eaten by termites. The respondent posits that
the two collectors of the Bureau of Customs never authenticated the xerox copies of the import entries;
instead, they only issued certifications stating therein the import entry numbers which were processed
by their office and the date the same were released.[60]

The respondent argues that it was not necessary for it to show the correct assessment, considering that
it is questioning the assessments not only because they are erroneous, but because they were issued
without factual basis and in patent violation of the assessment procedures laid down in the NIRC of
1977, as amended.[61] It is also pointed out that the petitioner failed to use the tax returns filed by the
respondent in computing the deficiency taxes which is contrary to law;[62] as such, the deficiency
assessments constituted deprivation of property without due process of law.[63]

Central to the second issue is Section 16 of the NIRC of 1977, as amended,[64] which provides that the
Commissioner of Internal Revenue has the power to make assessments and prescribe additional
requirements for tax administration and enforcement. Among such powers are those provided in
paragraph (b) thereof, which we quote:

(b) Failure to submit required returns, statements, reports and other documents. When a report
required by law as a basis for the assessment of any national internal revenue tax shall not be
forthcoming within the time fixed by law or regulation or when there is reason to believe that any such
report is false, incomplete or erroneous, the Commissioner shall assess the proper tax on the best
evidence obtainable.

In case a person fails to file a required return or other document at the time prescribed by law,
or willfully or otherwise files a false or fraudulent return or other document, the Commissioner shall
make or amend the return from his own knowledge and from such information as he can obtain through
testimony or otherwise, which shall be prima facie correct and sufficient for all legal purposes.[65]
This provision applies when the Commissioner of Internal Revenue undertakes to perform her
administrative duty of assessing the proper tax against a taxpayer, to make a return in case of a
taxpayers failure to file one, or to amend a return already filed in the BIR.

The petitioner may avail herself of the best evidence or other information or testimony by exercising her
power or authority under paragraphs (1) to (4) of Section 7 of the NIRC:

(1) To examine any book, paper, record or other data which may be relevant or material to such inquiry;

(2) To obtain information from any office or officer of the national and local governments, government
agencies or its instrumentalities, including the Central Bank of the Philippines and government owned or
controlled corporations;

(3) To summon the person liable for tax or required to file a return, or any officer or employee of such
person, or any person having possession, custody, or care of the books of accounts and other accounting
records containing entries relating to the business of the person liable for tax, or any other person, to
appear before the Commissioner or his duly authorized representative at a time and place specified in
the summons and to produce such books, papers, records, or other data, and to give testimony;

(4) To take such testimony of the person concerned, under oath, as may be relevant or material to such
inquiry; [66]

The best evidence envisaged in Section 16 of the 1977 NIRC, as amended, includes the corporate and
accounting records of the taxpayer who is the subject of the assessment process, the accounting records
of other taxpayers engaged in the same line of business, including their gross profit and net profit
sales.[67] Such evidence also includes data, record, paper, document or any evidence gathered by
internal revenue officers from other taxpayers who had personal transactions or from whom the subject
taxpayer received any income; and record, data, document and information secured from government
offices or agencies, such as the SEC, the Central Bank of the Philippines, the Bureau of Customs, and the
Tariff and Customs Commission.

The law allows the BIR access to all relevant or material records and data in the person of the taxpayer.
It places no limit or condition on the type or form of the medium by which the record subject to the
order of the BIR is kept. The purpose of the law is to enable the BIR to get at the taxpayers records in
whatever form they may be kept. Such records include computer tapes of the said records prepared by
the taxpayer in the course of business.[68] In this era of developing information-storage technology,
there is no valid reason to immunize companies with computer-based, record-keeping capabilities from
BIR scrutiny. The standard is not the form of the record but where it might shed light on the accuracy of
the taxpayers return.

In Campbell, Jr. v. Guetersloh,[69] the United States (U.S.) Court of Appeals (5th Circuit) declared that it
is the duty of the Commissioner of Internal Revenue to investigate any circumstance which led him to
believe that the taxpayer had taxable income larger than reported. Necessarily, this inquiry would have
to be outside of the books because they supported the return as filed. He may take the sworn testimony
of the taxpayer; he may take the testimony of third parties; he may examine and subpoena, if necessary,
traders and brokers accounts and books and the taxpayers book accounts. The Commissioner is not
bound to follow any set of patterns. The existence of unreported income may be shown by any
practicable proof that is available in the circumstances of the particular situation. Citing its ruling
in Kenney v. Commissioner,[70] the U.S. appellate court declared that where the records of the taxpayer
are manifestly inaccurate and incomplete, the Commissioner may look to other sources of information
to establish income made by the taxpayer during the years in question.[71]

We agree with the contention of the petitioner that the best evidence obtainable may consist of hearsay
evidence, such as the testimony of third parties or accounts or other records of other taxpayers similarly
circumstanced as the taxpayer subject of the investigation, hence, inadmissible in a regular proceeding
in the regular courts.[72] Moreover, the general rule is that administrative agencies such as the BIR are
not bound by the technical rules of evidence. It can accept documents which cannot be admitted in a
judicial proceeding where the Rules of Court are strictly observed. It can choose to give weight or
disregard such evidence, depending on its trustworthiness.

However, the best evidence obtainable under Section 16 of the 1977 NIRC, as amended, does not
include mere photocopies of records/documents. The petitioner, in making a preliminary and final tax
deficiency assessment against a taxpayer, cannot anchor the said assessment on mere machine copies
of records/documents. Mere photocopies of the Consumption Entries have no probative weight if
offered as proof of the contents thereof. The reason for this is that such copies are mere scraps of paper
and are of no probative value as basis for any deficiency income or business taxes against a taxpayer.
Indeed, in United States v. Davey,[73] the U.S. Court of Appeals (2nd Circuit) ruled that where the
accuracy of a taxpayers return is being checked, the government is entitled to use the original records
rather than be forced to accept purported copies which present the risk of error or tampering.[74]

In Collector of Internal Revenue v. Benipayo,[75] the Court ruled that the assessment must be based on
actual facts. The rule assumes more importance in this case since the xerox copies of the Consumption
Entries furnished by the informer of the EIIB were furnished by yet another informer. While the EIIB
tried to secure certified copies of the said entries from the Bureau of Customs, it was unable to do so
because the said entries were allegedly eaten by termites. The Court can only surmise why the EIIB or
the BIR, for that matter, failed to secure certified copies of the said entries from the Tariff and Customs
Commission or from the National Statistics Office which also had copies thereof. It bears stressing that
under Section 1306 of the Tariff and Customs Code, the Consumption Entries shall be the required
number of copies as prescribed by regulations.[76] The Consumption Entry is accomplished in
sextuplicate copies and quadruplicate copies in other places. In Manila, the six copies are distributed to
the Bureau of Customs, the Tariff and Customs Commission, the Declarant (Importer), the Terminal
Operator, and the Bureau of Internal Revenue. Inexplicably, the Commissioner and the BIR personnel
ignored the copy of the Consumption Entries filed with the BIR and relied on the photocopies supplied
by the informer of the EIIB who secured the same from another informer. The BIR, in preparing and
issuing its preliminary and final assessments against the respondent, even ignored the records on the
investigation made by the District Revenue officers on the respondents importations for 1987.
The original copies of the Consumption Entries were of prime importance to the BIR. This is so because
such entries are under oath and are presumed to be true and correct under penalty of falsification or
perjury. Admissions in the said entries of the importers documents are admissions against interest and
presumptively correct.[77]

In fine, then, the petitioner acted arbitrarily and capriciously in relying on and giving weight to the
machine copies of the Consumption Entries in fixing the tax deficiency assessments against the
respondent.

The rule is that in the absence of the accounting records of a taxpayer, his tax liability may be
determined by estimation. The petitioner is not required to compute such tax liabilities with
mathematical exactness. Approximation in the calculation of the taxes due is justified. To hold otherwise
would be tantamount to holding that skillful concealment is an invincible barrier to proof.[78] However,
the rule does not apply where the estimation is arrived at arbitrarily and capriciously.[79]

We agree with the contention of the petitioner that, as a general rule, tax assessments by tax examiners
are presumed correct and made in good faith. All presumptions are in favor of the correctness of a tax
assessment. It is to be presumed, however, that such assessment was based on sufficient evidence.
Upon the introduction of the assessment in evidence, a prima facie case of liability on the part of the
taxpayer is made.[80] If a taxpayer files a petition for review in the CTA and assails the assessment,
the prima facie presumption is that the assessment made by the BIR is correct, and that in preparing the
same, the BIR personnel regularly performed their duties. This rule for tax initiated suits is premised on
several factors other than the normal evidentiary rule imposing proof obligation on the petitioner-
taxpayer: the presumption of administrative regularity; the likelihood that the taxpayer will have access
to the relevant information; and the desirability of bolstering the record-keeping requirements of the
NIRC.[81]

However, the prima facie correctness of a tax assessment does not apply upon proof that an assessment
is utterly without foundation, meaning it is arbitrary and capricious. Where the BIR has come out with a
naked assessment, i.e., without any foundation character, the determination of the tax due is without
rational basis.[82] In such a situation, the U.S. Court of Appeals ruled[83] that the determination of the
Commissioner contained in a deficiency notice disappears. Hence, the determination by the CTA must
rest on all the evidence introduced and its ultimate determination must find support in credible
evidence.

The issue that now comes to fore is whether the tax deficiency assessment against the respondent
based on the certified copies of the Profit and Loss Statement submitted by the respondent to the SEC in
1987 and 1988, as well as certifications of Tomas and Danganan, is arbitrary, capricious and illegal. The
CTA ruled that the respondent failed to overcome the prima facie correctness of the tax deficiency
assessment issued by the petitioner, to wit:

The issue should be ruled in the affirmative as petitioner has failed to rebut the validity or correctness of
the aforementioned tax assessments. It is incongruous for petitioner to prove its cause by simply
drawing an inference unfavorable to the respondent by attacking the source documents (Consumption
Entries) which were the bases of the assessment and which were certified by the Chiefs of the Collection
Division, Manila International Container Port and the Port of Manila, as having been processed and
released in the name of the petitioner after payment of duties and taxes and the duly certified copies of
Financial Statements secured from the Securities and Exchange Commission. Any such inference cannot
operate to relieve petitioner from bearing its burden of proof and this Court has no warrant of
absolution. The Court should have been persuaded to grant the reliefs sought by the petitioner should it
have presented any evidence of relevance and competence required, like that of a certification from the
Bureau of Customs or from any other agencies, attesting to the fact that those consumption entries did
not really belong to them.

The burden of proof is on the taxpayer contesting the validity or correctness of an assessment to prove
not only that the Commissioner of Internal Revenue is wrong but the taxpayer is right (Tan Guan v. CTA,
19 SCRA 903), otherwise, the presumption in favor of the correctness of tax assessment stands (Sy Po v.
CTA, 164 SCRA 524). The burden of proving the illegality of the assessment lies upon the petitioner
alleging it to be so. In the case at bar, petitioner miserably failed to discharge this duty.[84]

We are not in full accord with the findings and ratiocination of the CTA. Based on the letter of the
petitioner to the respondent dated December 10, 1993, the tax deficiency assessment in question was
based on (a) the findings of the agents of the EIIB which was based, in turn, on the photocopies of the
Consumption Entries; (b) the Profit and Loss Statements of the respondent for 1987 and 1988; and (c)
the certifications of Tomas and Danganan dated August 7, 1990 and August 22, 1990:

In reply, please be informed that after a thorough evaluation of the attending facts, as well as the laws
and jurisprudence involved, this Office holds that you are liable to the assessed deficiency taxes. The
conclusion was arrived at based on the findings of agents of the Economic Intelligence & Investigation
Bureau (EIIB) and of our own examiners who have painstakingly examined the records furnished by the
Bureau of Customs and the Securities & Exchange Commission (SEC). The examination conducted
disclosed that while your actual sales for 1987 amounted to P110,731,559.00, you declared for taxation
purposes, as shown in the Profit and Loss Statements, the sum of P47,698,569.83 only. The difference,
therefore, of P63,032,989.17 constitutes as undeclared or unrecorded sales which must be subjected to
the income and sales taxes.

You also argued that our assessment has no basis since the alleged amount of underdeclared
importations were lifted from uncertified or unauthenticated xerox copies of consumption entries which
are not admissible in evidence. On this issue, it must be considered that in letters dated August 7 and
22, 1990, the Chief and Acting Chief of the Collection Division of the Manila International Container Port
and Port of Manila, respectively, certified that the enumerated consumption entries were filed,
processed and released from the port after payment of duties and taxes. It is noted that the certification
does not touch on the genuineness, authenticity and correctness of the consumption entries which are
all xerox copies, wherein the figures therein appearing may have been tampered which may render said
documents inadmissible in evidence, but for tax purposes, it has been held that the Commissioner is not
required to make his determination (assessment) on the basis of evidence legally admissible in a formal
proceeding in Court (Mertens, Vol. 9, p. 214, citing Cohen v. Commissioner). A statutory notice may be
based in whole or in part upon admissible evidence (Llorente v. Commissioner, 74 TC 260
(1980); Weimerskirch v. Commissioner, 67 TC 672 (1977); and Rosano v. Commissioner, 46 TC 681
(1966). In the case also of Weimerskirch v. Commissioner (1977), the assessment was given due course
in the presence of admissible evidence as to how the Commissioner arrived at his determination,
although there was no admissible evidence with respect to the substantial issue of whether the taxpayer
had unreported or undeclared income from narcotics sale. [85]

Based on a Memorandum dated October 23, 1990 of the IIPO, the source documents for the actual cost
of importation of the respondent are the machine copies of the Consumption Entries from the informer
which the IIPO claimed to have been certified by Tomas and Danganan:

The source documents for the total actual cost of importations, abovementioned, were the different
copies of Consumption Entries, Series of 1987, filed by subject with the Bureau of Customs, marked
Annexes F-1 to F-68. The total cost of importations is the sum of the Landed Costs and the Advance
Sales Tax as shown in the annexed entries. These entries were duly authenticated as having been
processed and released, after payment of the duties and taxes due thereon, by the Chief, Collection
Division, Manila International Container Port, dated August 7, 1990, Annex-G, and the Port of Manila,
dated August 22, 1990, Annex-H. So, it was established that subject-importations, mostly resins, really
belong to HANTEX TRADING CO., INC.[86]

It also appears on the worksheet of the IIPO, as culled from the photocopies of the Consumption Entries
from its informer, that the total cost of the respondents importation for 1987 was P105,761,527.00. Per
the report of Torres and Filamor, they also relied on the photocopies of the said Consumption Entries:

The importations made by taxpayer verified by us from the records of the Bureau of Customs and xerox
copies of which are hereto attached shows the big volume of importations made and not declared in the
income tax return filed by taxpayer.

Based on the above findings, it clearly shows that a prima facie case of fraud exists in the herein
transaction of the taxpayer, as a consequence of which, said transaction has not been possibly entered
into the books of accounts of the subject taxpayer.[87]

In fine, the petitioner based her finding that the 1987 importation of the respondent was underdeclared
in the amount of P105,761,527.00 on the worthless machine copies of the Consumption Entries. Aside
from such copies, the petitioner has no other evidence to prove that the respondent imported goods
costing P105,761,527.00. The petitioner cannot find solace on the certifications of Tomas and Danganan
because they did not authenticate the machine copies of the Consumption Entries, and merely indicated
therein the entry numbers of Consumption Entries and the dates when the Bureau of Customs released
the same. The certifications of Tomas and Danganan do not even contain the landed costs and the
advance sales taxes paid by the importer, if any. Comparing the certifications of Tomas and Danganan
and the machine copies of the Consumption Entries, only 36 of the entry numbers of such copies are
included in the said certifications; the entry numbers of the rest of the machine copies of the
Consumption Entries are not found therein.
Even if the Court would concede to the petitioners contention that the certification of Tomas and
Danganan authenticated the machine copies of the Consumption Entries referred to in the certification,
it appears that the total cost of importations inclusive of advance sales tax is only P64,324,953.00 far
from the amount of P105,716,527.00 arrived at by the EIIB and the BIR,[88] or even the amount
of P110,079,491.61 arrived at by Deputy Commissioner Deoferio, Jr.[89] As gleaned from the
certifications of Tomas and Danganan, the goods covered by the Consumption Entries were released by
the Bureau of Customs, from which it can be presumed that the respondent must have paid the taxes
due on the said importation. The petitioner did not adduce any documentary evidence to prove
otherwise.

Thus, the computations of the EIIB and the BIR on the quantity and costs of the importations of the
respondent in the amount of P105,761,527.00 for 1987 have no factual basis, hence, arbitrary and
capricious. The petitioner cannot rely on the presumption that she and the other employees of the BIR
had regularly performed their duties. As the Court held in Collector of Internal Revenue v.
Benipayo,[90] in order to stand judicial scrutiny, the assessment must be based on facts. The
presumption of the correctness of an assessment, being a mere presumption, cannot be made to rest on
another presumption.

Moreover, the uncontroverted fact is that the BIR District Revenue Office had repeatedly examined the
1987 books of accounts of the respondent showing its importations, and found that the latter had
minimal business tax liability. In this case, the presumption that the District Revenue officers performed
their duties in accordance with law shall apply. There is no evidence on record that the said officers
neglected to perform their duties as mandated by law; neither is there evidence aliunde that the
contents of the 1987 and 1988 Profit and Loss Statements submitted by the respondent with the SEC are
incorrect.

Admittedly, the respondent did not adduce evidence to prove its correct tax liability. However,
considering that it has been established that the petitioners assessment is barren of factual basis,
arbitrary and illegal, such failure on the part of the respondent cannot serve as a basis for a finding by
the Court that it is liable for the amount contained in the said assessment; otherwise, the Court would
thereby be committing a travesty.

On the disposition of the case, the Court has two options, namely, to deny the petition for lack of merit
and affirm the decision of the CA, without prejudice to the petitioners issuance of a new assessment
against the respondent based on credible evidence; or, to remand the case to the CTA for further
proceedings, to enable the petitioner to adduce in evidence certified true copies or duplicate original
copies of the Consumption Entries for the respondents 1987 importations, if there be any, and the
correct tax deficiency assessment thereon, without prejudice to the right of the respondent to adduce
controverting evidence, so that the matter may be resolved once and for all by the CTA. In the higher
interest of justice to both the parties, the Court has chosen the latter option. After all, as the Tax Court
of the United States emphasized in Harbin v. Commissioner of Internal Revenue,[91] taxation is not only
practical; it is vital. The obligation of good faith and fair dealing in carrying out its provision is reciprocal
and, as the government should never be over-reaching or tyrannical, neither should a taxpayer be
permitted to escape payment by the concealment of material facts.

IN LIGHT OF ALL THE FOREGOING, the petition is GRANTED. The Decision of the Court of Appeals is SET
ASIDE. The records are REMANDED to the Court of Tax Appeals for further proceedings, conformably
with the decision of this Court. No costs.

SO ORDERED.

Puno, (Chairman), Austria-Martinez, Tinga, and Chico-Nazario, JJ., concur.

DECISION

MENDOZA, J.:

This petition for review on certiorari seeks to set aside the May 17, 2007 Decision and the July 5, 2007
Resolution of the Court of Tax Appeals En Banc[1] (CTA-EB), in C.T.A. EB No. 90, affirming the October
26, 2004 Decision of the CTA-First Division[2] which, in turn, partially granted the petition for review of
respondent Sony Philippines, Inc. (Sony). The CTA-First Division decision cancelled the deficiency
assessment issued by petitioner Commissioner of Internal Revenue (CIR) against Sony for Value Added
Tax (VAT) but upheld the deficiency assessment for expanded withholding tax (EWT) in the amount
of P1,035,879.70 and the penalties for late remittance of internal revenue taxes in the amount
of P1,269, 593.90.[3]

THE FACTS:

On November 24, 1998, the CIR issued Letter of Authority No. 000019734 (LOA 19734) authorizing
certain revenue officers to examine Sonys books of accounts and other accounting records regarding
revenue taxes for the period 1997 and unverified prior years. On December 6, 1999, a preliminary
assessment for 1997 deficiency taxes and penalties was issued by the CIR which Sony
protested. Thereafter, acting on the protest, the CIR issued final assessment notices, the formal letter of
demand and the details of discrepancies.[4] Said details of the deficiency taxes and penalties for late
remittance of internal revenue taxes are as follows:

DEFICIENCY VALUE -ADDED TAX (VAT)

(Assessment No. ST-VAT-97-0124-2000)


Basic Tax Due P 7,958,700.00

Add: Penalties

Interest up to 3-31-2000 P 3,157,314.41

Compromise 25,000.00 3,182,314.41

Deficiency VAT Due P 11,141,014.41

DEFICIENCY EXPANDED WITHHOLDING TAX (EWT)

(Assessment No. ST-EWT-97-0125-2000)

Basic Tax Due P 1,416,976.90

Add: Penalties

Interest up to 3-31-2000 P 550,485.82

Compromise 25,000.00 575,485.82

Deficiency EWT Due P 1,992,462.72

DEFICIENCY OF VAT ON ROYALTY PAYMENTS

(Assessment No. ST-LR1-97-0126-2000)

Basic Tax Due P

Add: Penalties

Surcharge P 359,177.80

Interest up to 3-31-2000 87,580.34

Compromise 16,000.00 462,758.14

Penalties Due P 462,758.14


LATE REMITTANCE OF FINAL WITHHOLDING TAX

(Assessment No. ST-LR2-97-0127-2000)

Basic Tax Due P

Add: Penalties

Surcharge P 1,729,690.71

Interest up to 3-31-2000 508,783.07

Compromise 50,000.00 2,288,473.78

Penalties Due P 2,288,473.78

LATE REMITTANCE OF INCOME PAYMENTS

(Assessment No. ST-LR3-97-0128-2000)

Basic Tax Due P

Add: Penalties

25 % Surcharge P 8,865.34

Interest up to 3-31-2000 58.29

Compromise 2,000.00 10,923.60

Penalties Due P 10,923.60

GRAND TOTAL P 15,895,632.65[5]

Sony sought re-evaluation of the aforementioned assessment by filing a protest on February 2, 2000.
Sony submitted relevant documents in support of its protest on the 16th of that same month.[6]
On October 24, 2000, within 30 days after the lapse of 180 days from submission of the said supporting
documents to the CIR, Sony filed a petition for review before the CTA.[7]

After trial, the CTA-First Division disallowed the deficiency VAT assessment because the subsidized
advertising expense paid by Sony which was duly covered by a VAT invoice resulted in an input VAT
credit. As regards the EWT, the CTA-First Division maintained the deficiency EWT assessment on Sonys
motor vehicles and on professional fees paid to general professional partnerships. It also assessed the
amounts paid to sales agents as commissions with five percent (5%) EWT pursuant to Section 1(g) of
Revenue Regulations No. 6-85. The CTA-First Division, however, disallowed the EWT assessment on
rental expense since it found that the total rental deposit of P10,523,821.99 was incurred from January
to March 1998 which was again beyond the coverage of LOA 19734. Except for the compromise
penalties, the CTA-First Division also upheld the penalties for the late payment of VAT on royalties, for
late remittance of final withholding tax on royalty as of December 1997 and for the late remittance of
EWT by some of Sonys branches.[8] In sum, the CTA-First Division partly granted Sonys petition by
cancelling the deficiency VAT assessment but upheld a modified deficiency EWT assessment as well as
the penalties. Thus, the dispositive portion reads:

WHEREFORE, the petition for review is hereby PARTIALLY GRANTED. Respondent is ORDERED to CANCEL
and WITHDRAW the deficiency assessment for value-added tax for 1997 for lack of merit. However, the
deficiency assessments for expanded withholding tax and penalties for late remittance of internal
revenue taxes are UPHELD.

Accordingly, petitioner is DIRECTED to PAY the respondent the deficiency expanded withholding tax in
the amount of P1,035,879.70 and the following penalties for late remittance of internal revenue taxes in
the sum of P1,269,593.90:

1. VAT on Royalty P 429,242.07

2. Withholding Tax on Royalty 831,428.20

3. EWT of Petitioners Branches 8,923.63

Total P 1,269,593.90

Plus 20% delinquency interest from January 17, 2000 until fully paid pursuant to Section 249(C)(3) of the
1997 Tax Code.
SO ORDERED.[9]

The CIR sought a reconsideration of the above decision and submitted the following grounds in support
thereof:

A. The Honorable Court committed reversible error in holding that petitioner is not liable for the
deficiency VAT in the amount of P11,141,014.41;

B. The Honorable court committed reversible error in holding that the commission expense in the
amount of P2,894,797.00 should be subjected to 5% withholding tax instead of the 10% tax rate;

C. The Honorable Court committed a reversible error in holding that the withholding tax assessment
with respect to the 5% withholding tax on rental deposit in the amount of P10,523,821.99 should be
cancelled; and

D. The Honorable Court committed reversible error in holding that the remittance of final withholding
tax on royalties covering the period January to March 1998 was filed on time.[10]

On April 28, 2005, the CTA-First Division denied the motion for reconsideration. Unfazed, the CIR filed a
petition for review with the CTA-EB raising identical issues:

1. Whether or not respondent (Sony) is liable for the deficiency VAT in the amount of
P11,141,014.41;

2. Whether or not the commission expense in the amount of P2,894,797.00 should be subjected to
10% withholding tax instead of the 5% tax rate;
3. Whether or not the withholding assessment with respect to the 5% withholding tax on rental
deposit in the amount of P10,523,821.99 is proper; and

4. Whether or not the remittance of final withholding tax on royalties covering the period January to
March 1998 was filed outside of time.[11]

Finding no cogent reason to reverse the decision of the CTA-First Division, the CTA-EB dismissed CIRs
petition on May 17, 2007. CIRs motion for reconsideration was denied by the CTA-EB on July 5, 2007.

The CIR is now before this Court via this petition for review relying on the very same grounds it raised
before the CTA-First Division and the CTA-EB. The said grounds are reproduced below:

GROUNDS FOR THE ALLOWANCE OF THE PETITION

THE CTA EN BANC ERRED IN RULING THAT RESPONDENT IS NOT LIABLE FOR DEFICIENCY VAT IN THE
AMOUNT OF PHP11,141,014.41.

II

AS TO RESPONDENTS DEFICIENCY EXPANDED WITHHOLDING TAX IN THE AMOUNT OF PHP1,992,462.72:

A. THE CTA EN BANC ERRED IN RULING THAT THE COMMISSION EXPENSE IN THE AMOUNT OF
PHP2,894,797.00 SHOULD BE SUBJECTED TO A WITHHOLDING TAX OF 5% INSTEAD OF THE 10% TAX
RATE.

B. THE CTA EN BANC ERRED IN RULING THAT THE ASSESSMENT WITH RESPECT TO THE 5%
WITHHOLDING TAX ON RENTAL DEPOSIT IN THE AMOUNT OF PHP10,523,821.99 IS NOT PROPER.
III

THE CTA EN BANC ERRED IN RULING THAT THE FINAL WITHHOLDING TAX ON ROYALTIES COVERING THE
PERIOD JANUARY TO MARCH 1998 WAS FILED ON TIME.[12]

Upon filing of Sonys comment, the Court ordered the CIR to file its reply thereto. The CIR subsequently
filed a manifestation informing the Court that it would no longer file a reply. Thus, on December 3, 2008,
the Court resolved to give due course to the petition and to decide the case on the basis of the pleadings
filed.[13]

The Court finds no merit in the petition.

The CIR insists that LOA 19734, although it states the period 1997 and unverified prior years, should be
understood to mean the fiscal year ending in March 31, 1998.[14]The Court cannot agree.

Based on Section 13 of the Tax Code, a Letter of Authority or LOA is the authority given to the
appropriate revenue officer assigned to perform assessment functions. It empowers or enables said
revenue officer to examine the books of account and other accounting records of a taxpayer for the
purpose of collecting the correct amount of tax.[15]The very provision of the Tax Code that the CIR
relies on is unequivocal with regard to its power to grant authority to examine and assess a taxpayer.

SEC. 6. Power of the Commissioner to Make Assessments and Prescribe Additional Requirements for Tax
Administration and Enforcement.

(A)Examination of Returns and Determination of tax Due. After a return has been filed as required under
the provisions of this Code, the Commissioner or his duly authorized representative may authorize the
examination of any taxpayer and the assessment of the correct amount of tax: Provided, however, That
failure to file a return shall not prevent the Commissioner from authorizing the examination of any
taxpayer. x x x [Emphases supplied]
Clearly, there must be a grant of authority before any revenue officer can conduct an examination or
assessment. Equally important is that the revenue officer so authorized must not go beyond the
authority given. In the absence of such an authority, the assessment or examination is a nullity.

As earlier stated, LOA 19734 covered the period 1997 and unverified prior years. For said reason, the CIR
acting through its revenue officers went beyond the scope of their authority because the deficiency VAT
assessment they arrived at was based on records from January to March 1998 or using the fiscal year
which ended in March 31, 1998. As pointed out by the CTA-First Division in its April 28, 2005 Resolution,
the CIR knew which period should be covered by the investigation. Thus, if CIR wanted or intended the
investigation to include the year 1998, it should have done so by including it in the LOA or issuing
another LOA.

Upon review, the CTA-EB even added that the coverage of LOA 19734, particularly the phrase and
unverified prior years, violated Section C of Revenue Memorandum Order No. 43-90 dated September
20, 1990, the pertinent portion of which reads:

3. A Letter of Authority should cover a taxable period not exceeding one taxable year. The practice of
issuing L/As covering audit of unverified prior years is hereby prohibited. If the audit of a taxpayer shall
include more than one taxable period, the other periods or years shall be specifically indicated in the
L/A.[16] [Emphasis supplied]

On this point alone, the deficiency VAT assessment should have been disallowed. Be that as it may, the
CIRs argument, that Sonys advertising expense could not be considered as an input VAT credit because
the same was eventually reimbursed by Sony International Singapore (SIS), is also erroneous.

The CIR contends that since Sonys advertising expense was reimbursed by SIS, the former never incurred
any advertising expense. As a result, Sony is not entitled to a tax credit. At most, the CIR continues, the
said advertising expense should be for the account of SIS, and not Sony.[17]

The Court is not persuaded. As aptly found by the CTA-First Division and later affirmed by the CTA-EB,
Sonys deficiency VAT assessment stemmed from the CIRs disallowance of the input VAT credits that
should have been realized from the advertising expense of the latter.[18] It is evident under Section
110[19] of the 1997 Tax Code that an advertising expense duly covered by a VAT invoice is a legitimate
business expense. This is confirmed by no less than CIRs own witness, Revenue Officer Antonio
Aluquin.[20] There is also no denying that Sony incurred advertising expense. Aluquin testified that
advertising companies issued invoices in the name of Sony and the latter paid for the
same.[21] Indubitably, Sony incurred and paid for advertising expense/ services. Where the money came
from is another matter all together but will definitely not change said fact.

The CIR further argues that Sony itself admitted that the reimbursement from SIS was income and, thus,
taxable. In support of this, the CIR cited a portion of Sonys protest filed before it:

The fact that due to adverse economic conditions, Sony-Singapore has granted to our client a subsidy
equivalent to the latters advertising expenses will not affect the validity of the input taxes from such
expenses. Thus, at the most, this is an additional income of our client subject to income tax. We submit
further that our client is not subject to VAT on the subsidy income as this was not derived from the sale
of goods or services.[22]

Insofar as the above-mentioned subsidy may be considered as income and, therefore, subject to income
tax, the Court agrees. However, the Court does not agree that the same subsidy should be subject to the
10% VAT. To begin with, the said subsidy termed by the CIR as reimbursement was not even exclusively
earmarked for Sonys advertising expense for it was but an assistance or aid in view of Sonys dire or
adverse economic conditions, and was only equivalent to the latters (Sonys) advertising expenses.

Section 106 of the Tax Code explains when VAT may be imposed or exacted. Thus:

SEC. 106. Value-added Tax on Sale of Goods or Properties.

(A) Rate and Base of Tax. There shall be levied, assessed and collected on every sale, barter or exchange
of goods or properties, value-added tax equivalent to ten percent (10%) of the gross selling price or
gross value in money of the goods or properties sold, bartered or exchanged, such tax to be paid by the
seller or transferor.

Thus, there must be a sale, barter or exchange of goods or properties before any VAT may be levied.
Certainly, there was no such sale, barter or exchange in the subsidy given by SIS to Sony. It was but a
dole out by SIS and not in payment for goods or properties sold, bartered or exchanged by Sony.
In the case of CIR v. Court of Appeals (CA),[23] the Court had the occasion to rule that services rendered
for a fee even on reimbursement-on-cost basis only and without realizing profit are also subject to
VAT. The case, however, is not applicable to the present case. In that case, COMASERCO rendered
service to its affiliates and, in turn, the affiliates paid the former reimbursement-on-cost which means
that it was paid the cost or expense that it incurred although without profit. This is not true in the
present case. Sony did not render any service to SIS at all. The services rendered by the advertising
companies, paid for by Sony using SIS dole-out, were for Sony and not SIS. SIS just gave assistance to
Sony in the amount equivalent to the latters advertising expense but never received any goods,
properties or service from Sony.

Regarding the deficiency EWT assessment, more particularly Sonys commission expense, the CIR insists
that said deficiency EWT assessment is subject to the ten percent (10%) rate instead of the five percent
(5%) citing Revenue Regulation No. 2-98 dated April 17, 1998.[24] The said revenue regulation provides
that the 10% rate is applied when the recipient of the commission income is a natural person. According
to the CIR, Sonys schedule of Selling, General and Administrative expenses shows the commission
expense as commission/dealer salesman incentive, emphasizing the word salesman.

On the other hand, the application of the five percent (5%) rate by the CTA-First Division is based on
Section 1(g) of Revenue Regulations No. 6-85 which provides:

(g) Amounts paid to certain Brokers and Agents. On gross payments to customs, insurance, real estate
and commercial brokers and agents of professional entertainers five per centum (5%).[25]

In denying the very same argument of the CIR in its motion for reconsideration, the CTA-First Division,
held:

x x x, commission expense is indeed subject to 10% withholding tax but payments made to broker is
subject to 5% withholding tax pursuant to Section 1(g) of Revenue Regulations No. 6-85. While the
commission expense in the schedule of Selling, General and Administrative expenses submitted by
petitioner (SPI) to the BIR is captioned as commission/dealer salesman incentive the same does not
justify the automatic imposition of flat 10% rate. As itemized by petitioner, such expense is composed of
Commission Expense in the amount of P10,200.00 and Broker Dealer of P2,894,797.00.[26]
The Court agrees with the CTA-EB when it affirmed the CTA-First Division decision. Indeed, the
applicable rule is Revenue Regulations No. 6-85, as amended by Revenue Regulations No. 12-94, which
was the applicable rule during the subject period of examination and assessment as specified in the
LOA. Revenue Regulations No. 2-98, cited by the CIR, was only adopted in April 1998 and, therefore,
cannot be applied in the present case. Besides, the withholding tax on brokers and agents was only
increased to 10% much later or by the end of July 2001 under Revenue Regulations No. 6-2001.[27] Until
then, the rate was only 5%.

The Court also affirms the findings of both the CTA-First Division and the CTA-EB on the deficiency EWT
assessment on the rental deposit. According to their findings, Sony incurred the subject rental deposit in
the amount of P10,523,821.99 only from January to March 1998. As stated earlier, in the absence of the
appropriate LOA specifying the coverage, the CIRs deficiency EWT assessment from January to March
1998, is not valid and must be disallowed.

Finally, the Court now proceeds to the third ground relied upon by the CIR.

The CIR initially assessed Sony to be liable for penalties for belated remittance of its FWT on royalties (i)
as of December 1997; and (ii) for the period from January to March 1998. Again, the Court agrees with
the CTA-First Division when it upheld the CIR with respect to the royalties for December 1997 but
cancelled that from January to March 1998.

The CIR insists that under Section 3[28] of Revenue Regulations No. 5-82 and Sections 2.57.4 and
2.58(A)(2)(a)[29] of Revenue Regulations No. 2-98, Sony should also be made liable for the FWT on
royalties from January to March of 1998. At the same time, it downplays the relevance of the
Manufacturing License Agreement (MLA) between Sony and Sony-Japan, particularly in the payment of
royalties.

The above revenue regulations provide the manner of withholding remittance as well as the payment of
final tax on royalty. Based on the same, Sony is required to deduct and withhold final taxes on royalty
payments when the royalty is paid or is payable. After which, the corresponding return and remittance
must be made within 10 days after the end of each month. The question now is when does the royalty
become payable?

Under Article X(5) of the MLA between Sony and Sony-Japan, the following terms of royalty payments
were agreed upon:
(5)Within two (2) months following each semi-annual period ending June 30 and December 31, the
LICENSEE shall furnish to the LICENSOR a statement, certified by an officer of the LICENSEE, showing
quantities of the MODELS sold, leased or otherwise disposed of by the LICENSEE during such respective
semi-annual period and amount of royalty due pursuant this ARTICLE X therefore, and the LICENSEE
shall pay the royalty hereunder to the LICENSOR concurrently with the furnishing of the above
statement.[30]

Withal, Sony was to pay Sony-Japan royalty within two (2) months after every semi-annual period which
ends in June 30 and December 31. However, the CTA-First Division found that there was accrual of
royalty by the end of December 1997 as well as by the end of June 1998. Given this, the FWTs should
have been paid or remitted by Sony to the CIR on January 10, 1998 and July 10, 1998. Thus, it was
correct for the CTA-First Division and the CTA-EB in ruling that the FWT for the royalty from January to
March 1998 was seasonably filed. Although the royalty from January to March 1998 was well within the
semi-annual period ending June 30, which meant that the royalty may be payable until August 1998
pursuant to the MLA, the FWT for said royalty had to be paid on or before July 10, 1998 or 10 days from
its accrual at the end of June 1998. Thus, when Sony remitted the same on July 8, 1998, it was not yet
late.

In view of the foregoing, the Court finds no reason to disturb the findings of the CTA-EB.

WHEREFORE, the petition is DENIED.

SO ORDERED.

JOSE CATRAL MENDOZA

Associate Justice

WE CONCUR:
ANTONIO T. CARPIO

Associate Justice

Chairperson

TERESITA J. LEONARDO-DE CASTRO DIOSDADO M. PERALTA

Associate Justice Associate Justice

ROBERTO A. ABAD

Associate Justice

ATTESTATION

I attest that the conclusions in the above Decision had been reached in consultation before the case was
assigned to the writer of the opinion of the Courts Division.
ANTONIO T. CARPIO

Associate Justice

Chairperson, Second Division

CERTIFICATION

Pursuant to Section 13, Article VIII of the Constitution and the Division Chairpersons Attestation, I certify
that the conclusions in the above Decision had been reached in consultation before the case was
assigned to the writer of the opinion of the Courts Division.

G.R. No. L-36181 October 23, 1982

MERALCO SECURITIES CORPORATION (now FIRST PHILIPPINE HOLDINGS CORPORATION), petitioner,


vs.
HON. VICTORINO SAVELLANO and ASUNCION BARON VDA. DE MANIAGO, et al., as heirs of the late Juan
G. Maniago, respondents.

G.R. No. L-36748 October 23, 1982

COMMISSIONER OF INTERNAL REVENUE, petitioner,


vs.
HON. VICTORINO SAVELLANO and ASUNCION BARON VDA. DE MANIAGO, et al., as heirs of the late Juan
G. Maniago, respondents.

G.R. No. L-36181

San Juan, Africa, Gonzales & San Agustin for petitioner.

Ramon A. Gonzales for respondents.

TEEHANKEE, J.:

These are original actions for certiorari to set aside and annul the writ of mandamus issued by Judge
Victorino A. Savellano of the Court of First Instance of Manila in Civil Case No. 80830 ordering petitioner
Meralco Securities Corporation (now First Philippine Holdings Corporation) to pay, and petitioner
Commissioner of Internal Revenue to collect from the former, the amount of P51,840,612.00, by way of
alleged deficiency corporate income tax, plus interests and surcharges due thereon and to pay private
respondents 25% of the total amount collectible as informer's reward.

On May 22, 1967, the late Juan G. Maniago (substituted in these proceedings by his wife and children)
submitted to petitioner Commissioner of Internal Revenue confidential denunciation against the
Meralco Securities Corporation for tax evasion for having paid income tax only on 25 % of the dividends
it received from the Manila Electric Co. for the years 1962-1966, thereby allegedly shortchanging the
government of income tax due from 75% of the said dividends.

Petitioner Commissioner of Internal Revenue caused the investigation of the denunciation after which
he found and held that no deficiency corporate income tax was due from the Meralco Securities
Corporation on the dividends it received from the Manila Electric Co., since under the law then
prevailing (section 24[a] of the National Internal Revenue Code) "in the case of dividends received by a
domestic or foreign resident corporation liable to (corporate income) tax under this Chapter . . . .only
twenty-five per centum thereof shall be returnable for the purposes of the tax imposed under this
section." The Commissioner accordingly rejected Maniago's contention that the Meralco from whom the
dividends were received is "not a domestic corporation liable to tax under this Chapter." In a letter
dated April 5, 1968, the Commissioner informed Maniago of his findings and ruling and therefore denied
Maniago's claim for informer's reward on a non-existent deficiency. This action of the Commissioner was
sustained by the Secretary of Finance in a 4th Indorsement dated May 11, 1971.

On August 28, 1970, Maniago filed a petition for mandamus, and subsequently an amended petition for
mandamus, in the Court of First Instance of Manila, docketed therein as Civil Case No. 80830, against
the Commissioner of Internal Revenue and the Meralco Securities Corporation to compel the
Commissioner to impose the alleged deficiency tax assessment on the Meralco Securities Corporation
and to award to him the corresponding informer's reward under the provisions of R.A. 2338.

On October 28, 1978, the Commissioner filed a motion to dismiss, arguing that since in matters of
issuance and non-issuance of assessments, he is clothed under the National Internal Revenue Code and
existing rules and regulations with discretionary power in evaluating the facts of a case and since
mandamus win not lie to compel the performance of a discretionary power, he cannot be compelled to
impose the alleged tax deficiency assessment against the Meralco Securities Corporation. He further
argued that mandamus may not lie against him for that would be tantamount to a usurpation of
executive powers, since the Office of the Commissioner of Internal Revenue is undeniably under the
control of the executive department.

On the other hand, the Meralco Securities Corporation filed its answer, dated January 15, 1971,
interposing as special and/or affirmative defenses that the petition states no cause of action, that the
action is premature, that mandamus win not lie to compel the Commissioner of Internal Revenue to
make an assessment and/or effect the collection of taxes upon a taxpayer, that since no taxes have
actually been recovered and/or collected, Maniago has no right to recover the reward prayed for, that
the action of petitioner had already prescribed and that respondent court has no jurisdiction over the
subject matter as set forth in the petition, the same being cognizable only by the Court of Tax Appeals.
On January 10, 1973, the respondent judge rendered a decision granting the writ prayed for and
ordering the Commissioner of Internal Revenue to assess and collect from the Meralco Securities
Corporation the sum of P51,840,612.00 as deficiency corporate income tax for the period 1962 to 1969
plus interests and surcharges due thereon and to pay 25% thereof to Maniago as informer's reward.

All parties filed motions for reconsideration of the decision but the same were denied by respondent
judge in his order dated April 6, 1973, with respondent judge denying respondents' claim for attorneys
fees and for execution of the decision pending appeal.

Hence, the Commissioner filed a separate petition with this Court, docketed as G.R. No. L-36748 praying
that the decision of respondent judge dated January 10, 1973 and his order dated April 6, 1973 be
reconsidered for respondent judge has no jurisdiction over the subject matter of the case and that the
issuance or non-issuance of a deficiency assessment is a prerogative of the Commissioner of Internal
Revenue not reviewable by mandamus.

The Meralco Securities Corporation (now First Philippine Holdings Corporation) likewise appealed the
same decision of respondent judge in G.R. No. L-36181 and in the Court's resolution dated June 13,
1973, the two cases were ordered consolidated.

We grant the petitions.

Respondent judge has no jurisdiction to take cognizance of the case because the subject matter thereof
clearly falls within the scope of cases now exclusively within the jurisdiction of the Court of Tax Appeals.
Section 7 of Republic Act No. 1125, enacted June 16, 1954, granted to the Court of Tax Appeals exclusive
appellate jurisdiction to review by appeal, among others, decisions of the Commissioner of Internal
Revenue in cases involving disputed assessments, refunds of internal revenue taxes, fees or other
charges, penalties imposed in relation thereto, or other matters arising under the National Internal
Revenue Code or other law or part of law administered by the Bureau of Internal Revenue. The law
transferred to the Court of Tax Appeals jurisdiction over all cases involving said assessments previously
cognizable by courts of first instance, and even those already pending in said courts. 1 The question of
whether or not to impose a deficiency tax assessment on Meralco Securities Corporation undoubtedly
comes within the purview of the words "disputed assessments" or of "other matters arising under the
National Internal Revenue Code . . . .In the case of Blaquera vs. Rodriguez, et al, 2 this Court ruled that
"the determination of the correctness or incorrectness of a tax assessment to which the taxpayer is not
agreeable, falls within the jurisdiction of the Court of Tax Appeals and not of the Court of First Instance,
for under the provisions of Section 7 of Republic Act No. 1125, the Court of Tax Appeals
has exclusive appellate jurisdiction to review, on appeal, any decision of the Collector of Internal
Revenue in cases involving disputed assessments and other matters arising under the National Internal
Revenue Code or other law or part of law administered by the Bureau of Internal Revenue."

Thus, even assuming arguendo that the right granted the taxpayers affected to question and appeal
disputed assessments, under section 7 of Republic Act No. 1125, may be availed of by strangers or
informers like the late Maniago, the most that he could have done was to appeal to the Court of Tax
Appeals the ruling of petitioner Commissioner of Internal Revenue within thirty (30) days from receipt
thereof pursuant to section 11 of Republic Act No. 1125. 3 He failed to take such an appeal to the tax
court. The ruling is clearly final and no longer subject to review by the courts. 4

It is furthermore a well-recognized rule that mandamus only lies to enforce the performance of a
ministerial act or duty 5 and not to control the performance of a discretionary power. 6 Purely
administrative and discretionary functions may not be interfered with by the courts. 7 Discretion, as
thus intended, means the power or right conferred upon the office by law of acting officially under
certain circumstances according to the dictates of his own judgment and conscience and not controlled
by the judgment or conscience of others. 8 mandamus may not be resorted to so as to interfere with the
manner in which the discretion shall be exercised or to influence or coerce a particular determination. 9

In an analogous case, where a petitioner sought to compel the Rehabilitation Finance Corporation to
accept payment of the balance of his indebtedness with his backpay certificates, the Court ruled that
"mandamus does not compel the Rehabilitation Finance Corporation to accept backpay certificates in
payment of outstanding loans. Although there is no provision expressly authorizing such acceptance, nor
is there one prohibiting it, yet the duty imposed by the Backpay Law upon said corporation as to the
acceptance or discount of backpay certificates is neither clear nor ministerial, but discretionary merely,
and such special civil action does not issue to control the exercise of discretion of a public
officer." 10 Likewise, we have held that courts have no power to order the Commissioner of Customs to
confiscate goods imported in violation of the Import Control Law, R.A. 426, as said forfeiture is subject
to the discretion of the said official, 11 nor may courts control the determination of whether or not an
applicant for a visa has a non-immigrant status or whether his entry into this country would be contrary
to public safety for it is not a simple ministerial function but an exercise of discretion. 12

Moreover, since the office of the Commissioner of Internal Revenue is charged with the administration
of revenue laws, which is the primary responsibility of the executive branch of the government,
mandamus may not he against the Commissioner to compel him to impose a tax assessment not found
by him to be due or proper for that would be tantamount to a usurpation of executive functions. As we
held in the case of Commissioner of Immigration vs. Arca 13 anent this principle, "the administration of
immigration laws is the primary responsibility of the executive branch of the government. Extensions of
stay of aliens are discretionary on the part of immigration authorities, and neither a petition for
mandamus nor one for certiorari can compel the Commissioner of Immigration to extend the stay of an
alien whose period to stay has expired.

Such discretionary power vested in the proper executive official, in the absence of arbitrariness or grave
abuse so as to go beyond the statutory authority, is not subject to the contrary judgment or control of
others. " "Discretion," when applied to public functionaries, means a power or right conferred upon
them by law of acting officially, under certain circumstances, uncontrolled by the judgment or
consciences of others. A purely ministerial act or duty in contradiction to a discretional act is one which
an officer or tribunal performs in a given state of facts, in a prescribed manner, in obedience to the
mandate of a legal authority, without regard to or the exercise of his own judgment upon the propriety
or impropriety of the act done. If the law imposes a duty upon a public officer and gives him the right to
decide how or when the duty shall be performed, such duty is discretionary and not ministerial. The
duty is ministerial only when the discharge of the same requires neither the exercise of official
discretion or judgment." 14

Thus, after the Commissioner who is specifically charged by law with the task of enforcing and
implementing the tax laws and the collection of taxes had after a mature and thorough study rendered
his decision or ruling that no tax is due or collectible, and his decision is sustained by the Secretary, now
Minister of Finance (whose act is that of the President unless reprobated), such decision or ruling is a
valid exercise of discretion in the performance of official duty and cannot be controlled much less
reversed by mandamus. A contrary view, whereby any stranger or informer would be allowed to usurp
and control the official functions of the Commissioner of Internal Revenue would create disorder and
confusion, if not chaos and total disruption of the operations of the government.

Considering then that respondent judge may not order by mandamus the Commissioner to issue the
assessment against Meralco Securities Corporation when no such assessment has been found to be due,
no deficiency taxes may therefore be assessed and collected against the said corporation. Since no taxes
are to be collected, no informer's reward is due to private respondents as the informer's heirs.
Informer's reward is contingent upon the payment and collection of unpaid or deficiency taxes. An
informer is entitled by way of reward only to a percentage of the taxes actually assessed and collected.
Since no assessment, much less any collection, has been made in the instant case, respondent judge's
writ for the Commissioner to pay respondents 25% informer's reward is gross error and without factual
nor legal basis.

WHEREFORE, the petitions are hereby granted and the questioned decision of respondent judge dated
January 10, 1973 and order dated April 6, 1973 are hereby reversed and set aside. With costs against
private respondents.

Melencio-Herrera, Plana, Vasquez and Relova, JJ., concur.

Gutierrez, Jr., J., took no part.

G.R. No. 81446 August 18, 1988

BONIFACIA SY PO, petitioner,


vs.
HONORABLE COURT OF TAX APPEALS AND HONORABLE COMMISSIONER OF INTERNAL
REVENUE, respondents.

Basilio E. Duaban for petitioner.

SARMIENTO, J.:

This is an appeal from the decision 1 of the respondent Court of Tax Appeals, dated September 30,1987,
which affirmed an earlier decision of the correspondent Commissioner of Internal Revenue in
assessment letters dated August 16, 1972 and September 26, 1972, which ordered the payment by the
petitioner of deficiency income tax for 1966 to 1970 in the amount of P7,154,685.16 and deficiency
specific tax for January 2, 1964 to January 19, 1972, in the amount of P5,595,003.68.

We adopt the respondent court's finding of facts, to wit:

Petitioner is the widow of the late Mr. Po Bien Sing who died on September 7, 1980. In the taxable years
1964 to 1972, the deceased Po Bien Sing was the sole proprietor of Silver Cup Wine Factory (Silver Cup
for brevity), Talisay, Cebu. He was engaged in the business of manufacture and sale of compounded
liquors, using alcohol and other ingredients as raw materials.

On the basis of a denunciation against Silver Cup allegedly "for tax evasion amounting to millions of
pesos" the then Secretary of Finance Cesar Virata directed the Finance-BIR--NBI team constituted under
Finance Department Order No. 13-70 dated February 19, 1971 (Exh- 3, pp. 532-553, Folder II, BIR rec.) to
conduct the corresponding investigation in a memorandum dated April 2, 1971 (p. 528, Folder II, BIR
rec.). Accordingly, a letter and a subpoena duces tecum dated April 13,1971 and May 3,1971,
respectively, were issued against Silver Cup requesting production of the accounting records and other
related documents for the examination of the team. (Exh. 11, pp. 525-526, Folder II, BIR rec.). Mr. Po
Bien Sing did not produce his books of accounts as requested (Affidavit dated December 24, 1971 of Mr.
Generoso. Quinain of the team, p. 525, Folder H, BIR rec.). This prompted the team with the assistance
of the PC Company, Cebu City, to enter the factory bodega of Silver Cup and seized different brands,
consisting of 1,555 cases of alcohol products. (Exh. 22, Memorandum Report of the Team dated June 5,
1971, pp. 491-492, Folder II, BIR rec.). The inventory lists of the seized alcohol products are contained in
Volumes I, II, III, IV and V (Exhibits 14, 15, 16, 17, and 18, respectively, BIR rec.). On the basis of the
team's report of investigation, the respondent Commissioner of Internal Revenue assessed Mr. Po Bien
Sing deficiency income tax for 1966 to 1970 in the amount of P7,154,685.16 (Exh. 6 pp. 17-19, Folder I,
BIR rec.) and for deficiency specific tax for January 2,1964 to January 19, 1972 in the amount of
P5,595,003.68 (Exh. 8, p. 107, Folder I, BIR rec.).

Petitioner protested the deficiency assessments through letters dated October 9 and October 30, 1972
(Exhs. 7 and 9, pp. 27-28; pp. 152-159, respectively, BIR rec.), which protests were referred for
reinvestigation. The corresponding report dated August 13, 1981 (Exh. 1 0, pp. 355, Folder I, BIR rec.)
recommended the reiteration of the assessments in view of the taxpayer's persistent failure to present
the books of accounts for examination (Exh. 8, p. 107, Folder I, BIR rec.), compelling respondent to issue
warrants of distraint and levy on September 10, 1981 (Exh. 11, p. 361, Folder I, BIR rec.).

The warrants were admittedly received by petitioner on October 14, 1981 (Par. IX, Petition; admitted
par. 2, Answer), which petitioner deemed respondent's decision denying her protest on the subject
assessments. Hence, petitioner's appeal on October 29,1981. 2

The petitioner assigns the following errors:

I
RESPONDENT INTENTIONALLY ERRED IN HOLDING THAT PETITIONER HAS NOT PRESENTED ANY
EVIDENCE OF RELEVANCE AND COMPETENCE REQUIRED TO BASH THE TROUBLING DISCREPANCIES AND
SQUARE THE ISSUE OF ILLEGALITY POSITED ON THE SUBJECT ASSESSMENTS.

II

RESPONDENT COURT OF TAX APPEALS PALPABLY ERRED IN DECIDING THE CASE IN A WAY CONTRARY TO
THE DOCTRINES ALREADY LAID DOWN BY THIS COURT.

III

RESPONDENT COURT OF TAX APPEALS GRAVELY ERRED IN FINDING PO BEEN SING TO HAVE INCURRED
THE ALLEGED DEFICIENCY TAXES IN QUESTION. 3

We affirm.

Settled is the rule that the factual findings of the Court of Tax Appeals are binding upon this Honorable
Court and can only be disturbed on appeal if not supported by substantial evidence.4

The assignments of errors boils down to a single issue previously raised before the respondent
Court, i.e., whether or not the assessments have valid and legal bases.

The applicable legal provision is Section 16(b) of the National Internal Revenue Code of 1977 as
amended. It reads:

Sec. 16. Power of the Commissioner of Internal Revenue to make assessments.—

xxx xxx xxx

(b) Failure to submit required returns, statements, reports and other documents. - When a report
required by law as a basis for the assessment of an national internal revenue tax shall not be
forthcoming within the time fixed by law or regulation or when there is reason to believe that any such
report is false, incomplete, or erroneous, the Commissioner of Internal Revenue shall assess the proper
tax on the best evidence obtainable.

In case a person fails to file a required return or other document at the time prescribed by law, or
willfully or otherwise, files a false or fraudulent return or other documents, the Commissioner shall
make or amend the return from his own knowledge and from such information as he can obtain through
testimony or otherwise, which shall be prima facie correct and sufficient for all legal purposes.

The law is specific and clear. The rule on the "best evidence obtainable" applies when a tax report
required by law for the purpose of assessment is not available or when the tax report is incomplete or
fraudulent.

In the instant case, the persistent failure of the late Po Bien Sing and the herein petitioner to present
their books of accounts for examination for the taxable years involved left the Commissioner of Internal
Revenue no other legal option except to resort to the power conferred upon him under Section 16 of
the Tax Code.

The tax figures arrived at by the Commissioner of Internal Revenue are by no means arbitrary. We
reproduce the respondent court's findings, to wit:

As thus shown, on the basis of the quantity of bottles of wines seized during the raid and the sworn
statements of former employees Messrs. Nelson S. Po and Alfonso Po taken on May 26, and 27,1971,
respectively, by the investigating team in Cebu City (Exhs. 4 and 5, pp. 514-517, pp. 511-513, Folder 11,
BIR rec.), it was ascertained that the Silver Cup for the years 1964 to 1970, inclusive, utilized and
consumed in the manufacture of compounded liquours and other products 20,105 drums of alcohol as
raw materials 81,288,787 proof liters of alcohol. As determined, the total specific tax liability of the
taxpayer for 1964 to 1971 amounted to P5,593,003.68 (Exh. E, petition, p. 10, CTA rec.)

Likewise, the team found due from Silver Cup deficiency income taxes for the years 1966 to 1970
inclusive in the aggregate sum of P7,154,685.16, as follows:

1966 P207,636.24

1967 645,335.04

1968 1,683,588.48

1969 1,589,622.48

1970 3,028,502.92

Total amount due.

and collectible P7,154,685.16

The 50% surcharge has been imposed, pursuant to Section 72 * of the Tax Code and tax 1/2% monthly
interest has likewise been imposed pursuant to the provision of Section 51(d) ** of the Tax Code (Exh.
O, petition). 5

The petitioner assails these assessments as wrong.

In the case of Collector of Internal Revenue vs. Reyes, 6 we ruled:

Where the taxpayer is appealing to the tax court on the ground that the Collector's assessment is
erroneous, it is incumbent upon him to prove there what is the correct and just liability by a full and fair
disclosure of all pertinent data in his possession. Otherwise, if the taxpayer confines himself to proving
that the tax assessment is wrong, the tax court proceedings would settle nothing, and the way would be
left open for subsequent assessments and appeals in interminable succession.
Tax assessments by tax examiners are presumed correct and made in good faith. The taxpayer has the
duty to prove otherwise. 7 In the absence of proof of any irregularities in the performance of duties, an
assessment duly made by a Bureau of Internal Revenue examiner and approved by his superior officers
will not be disturbed. 8 All presumptions are in favor of the correctness of tax assessments. 9

On the whole, we find that the fraudulent acts detailed in the decision under review had not been
satisfactorily rebutted by the petitioner. There are indeed clear indications on the part of the taxpayer
to deprive the Government of the taxes due. The Assistant Factory Superintendent of Silver Cup, Nelson
Po gave the following testimony:

Annexes "A", "A-1 " to "A-17" show that from January to December 1970, Silver Cup had used in
production 189 drums of untaxed distilled alcohol and 3,722 drums of untaxed distilled alcohol. Can you
tell us how could this be possible with the presence of a revenue inspector in the premises of Silver Cup
during working hours?

Actually, the revenue inspector or storekeeper comes around once a week on the average. Sometimes,
when the storekeeper is around in the morning and Po Bein Sing wants to operate with untaxed alcohol
as raw materials, Po Bien Sing tells the storekeeper to go home because the factory is not going to
operate for the day. After the storekeeper leaves, the illegal operation then begins. Untaxed alcohol is
brought in from Cebu Alcohol Plant into the compound of Silver Cup sometimes at about 6:00 A.M. or at
12:00 noon or in the evening or even at mid-night when the storekeeper is not around. When the
storekeeper comes, he sees nothing because untaxed alcohol is brought directly to, and stored at, a
secret tunnel within the bodega itself inside the compound of Silver Cup.

In the same vein, the factory personnel manager testified that false entries were entered in the official
register book: thus,

A — As factory personnel manager and all-around handy man of Po Bien Sing, owner of Silver Cup, these
labels were entrusted to me to make the false entries in the official register book of Silver Cup, which I
did under the direction of Po Bien Sing. (Sworn statement, p. 512, Folder II, BIR rec.) 10 (Emphasis ours)

The existence of fraud as found by the respondents can not be lightly set aside absent substantial
evidence presented by the petitioner to counteract such finding. The findings of fact of the respondent
Court of Tax Appeals are entitled to the highest respect.11 We do not find anything in the questioned
decision that should disturb this long-established doctrine.

WHEREFORE, the Petition is DENIED. The Decision of the respondent Court of Tax Appeals is hereby
AFFIRMED. Costs against the petitioner.

SO ORDERED.

Melencio-Herrera, Paras and Padilla, JJ., concur.


Footnotes

1 Penned by Associate Judge Alex Z. Reyes and concurred in by Presiding Judge Amante Filler; Associate
Judge Constante C. Roaquin was on leave.

2 Rollo, Decision, 10-13.

3 Id., Petition, 3; Rollo, 4.

4 Aznar vs. CTA, L-20569, August 23,1974,58 SCRA 51 9; Manila Wine Merchants vs. Commissioner of
Internal Revenue, I,26145, February 20, 1984, 127 SCRA 483; La Suerte Cigar and Cigarette Factory vs.
Court of Tax Appeals, L-36130 and Alhambra Industries va. Court of Tax Appeals, L-36131, January
17,1985,134 SCRA 29.

* Old rule, Section 72 of the National Internal Revenue Code otherwise known as Commonwealth Act
No. 466:

Surcharges for failure to render returns and for rendering false and fraudulent returns.-The
Commissioner of Internal Revenue shall assess all income taxes. In case of willful neglect to file the
return or list within the time prescribed by law, or in case a false or fraudulent return or list is willingly
made, the Commissioner of Internal Revenue shall add to the tax or to the deficiency fax, in case any
payment has been made on the basis of such return before the discovery of the falsity of fraud, a
surcharge of fifty per centum of the amount of such tax or deficiency....

Now Section 248(b) of the National Internal Revenue Code of 1977 as amended:

In case of willful neglect to file the return within the period prescribed by this Code or regulations, or in
case a false or fraudulent return is willfully made, the penalty to be imposes shall be fifty percent (50%)
of the tax or of the deficiency tax, in case any payment has been made on the basis of such return
before the discovery of the falsity or fraud.

** Old Rule, Section 51(d) of the National Internal Revenue Code otherwise known as Commonwealth
Act No. 466:

Interest on deficiency.—Interest upon the amount determined as a deficiency shall be assessed at the
same time as the deficiency and shall be paid upon notice and demand from the Commissioner, of
Internal Revenue; and shall be collected as a part of the tax, at the rate of six per centum per
annumfrom the date prescribed for the payment of the tax (or, if the tax is paid in installments, from the
date prescribed for the payment of the first installment) to the date of the deficiency is
assessed: Provided, That the maximum amount that may be collected as interest on deficiency shall in
no case exceed the amount corresponding to a period of three years, the present provisions regarding
prescription to the contrary notwithstanding.

Now Section 249(a) of the National Internal Revenue Code of 1977 as amended:
In general—There shall be assessed and collected on any unpaid amount of tax, interest at the rate of
twenty percent (20%) per annum, or such higher rate as may be prescribed for payment until the
amount is fully paid.

5 Rollo, 14-15.

6 104 Phil. 1061 (1958) Unrep., Nos. L-11534 and L-11558, November 25, 1958.

7 Commissioner of Internal Revenue vs. Construction Resources of Asia, Inc., L-68230, November 25,
1986, 145 SCRA 671.

8 Gutierrez vs. Villegas, L-17117, July 31, 1963, 8 SCRA 527.

9 Collector of Internal Revenue vs. Bohol Land Transportation Co., L-13099 and L-13462, April
29,1960,58 O.G. 2407.

10 Rollo, Decision, 15-16.

11 Raymundo vs. Joya, L-27733, December 3, 1980, 101 SCRA 495; Sanchez vs. Commissioner, 102 Phil.
37 (1957); Commissioner vs. Priscilla Estate, 120 Phil. 125 (1964); Commissioner of Internal Revenue vs.
Ayala Securities Corporation, L-29485, March 31, 1976, 70 SCRA 204.

You might also like